Download as pdf or txt
Download as pdf or txt
You are on page 1of 56

AAOS 2009 Spine Self-Assessment Examination

2
AAOS 2009 Spine Self-Assessment Examination
1. Which is the best initial study for the diagnostic evaluation of diskogenic low
back pain?

1- MRI
2- Diskography
3- CT-diskography
4- Radiography
5- CT

DISCUSSION: Radiography is the best initial study for the evaluation of diskogenic low back
pain. The normal degenerative process can be evaluated. Vacuum phenomenon may be
found within the disk space. Other possible sources for back pain should also be evaluated.
The other tests may be beneficial but represent later imaging options. PREFE RESPONSE: 4

2. A patient who is an observant Jehovah’s Witness requires major surgery for


scoliosis that will likely result in significant blood loss. Which of the following might
the patient consider allowing the surgical team to use?

1- Transfusion of whole blood


2- Transfusion of packed red blood cells
3- A cell saver with continuity maintained in a “closed circuit”
4- Transfusion of plasma
5- Transfusion of platelets

DISCUSSION: Jehovah’s Witnesses will not accept the transfusion of blood or blood
products such as packed red or white cells, platelets, or plasma. However, many Jehovah’s
Witnesses will accept the use of a cell saver in a “closed circuit.” PREFERRED RESPON: 3

3. Which of the following is a contraindication to laminoplasty in a patient with


cervical spondylotic myelopathy?

1- Space available for the cord of less than 8 mm


2- Ossification of the posterior longitudinal ligament
3- Fixed cervical kyphosis
4- Previous posterior surgery
5- Concomitant cervical radiculopathy

DISCUSSION: Laminoplasty or any posterior decompressive procedure is contraindicated in


patients with cervical spondylotic myelopathy and cervical kyphosis. The residual kyphotic
posture of the cervical spine results in persistent spinal cord compression. The other choices
are not contraindications for laminoplasty. Concomitant cervical radiculopathy can be
addressed at the time of laminoplasty with a keyhole foraminotomy.
PREFERRED RESPONSE: 3
3
AAOS 2009 Spine Self-Assessment Examination
4. A 44-year-old man reports persistent left leg pain following a L5-S1
hemilaminotomy and partial diskectomy. Examination shows a grade 4 weakness of
the left extensor hallucis longus and a positive left straight leg raise. A radiograph is
shown in Figure 1a, and sagittal and axial MRI scans are shown in Figures 1b and 1c.
Nonsurgical management consisting of medication, physical therapy, and injections
has failed to provide relief. Surgical management should consist of

1- revision L5-S1 hemilaminotomy.


2- L5-S1 total disk arthroplasty.
3- L5 Gill laminectomy.
4- posterior foraminal decompression and fusion at L5-S1 with
instrumentation and bone graft.
5- stand-alone posterior lumbar interbody fusion.

DISCUSSION: The patient has a grade I isthmic spondylolisthesis at L5-S1. He has an L5


radiculopathy with foraminal stenosis. Any further treatment needs to include an arthrodesis
and foraminal decompression. Isolated interbody fusion is contraindicated in patients with
spondylolisthesis, as is total disk arthroplasty. Therefore, the best procedure is a posterior
fusion with instrumentation and bone graft along with a foraminal decompression.
PREFERRED RESPONSE: 4

5. Bisphosphonates are indicated in the treatment of osteoporosis in patients


who have a DEXA T-score of

1- between 0 and 1.
2- between 0 and -1.
3- -3.5 and are already on teriparatide.
4- within one standard deviation from the mean.
5- less than -1.

DISCUSSION: Bisphosphonates are indicated in the treatment of osteoporosis. They have


been shown to reduce the incidence of vertebral and extremity fractures in patients with a T-
score of less than -1. PREFERRED RESPONSE: 5

4
AAOS 2009 Spine Self-Assessment Examination
6. A 45-year-old man reports that he awoke 2 weeks ago with severe pain in his
right arm. Examination reveals weakness in the biceps, brachialis, and wrist
extensors. There is decreased sensation in the thumb and index finger and a
diminished brachioradialis reflex. Assuming this patient has a posterolateral
herniated nucleus pulposus, what level is involved?

1- C2-3
2- C3-4
3- C4-5
4- C5-6
5- C6-7

DISCUSSION: This is a classic C6 nerve injury, and it is most likely the result of a herniated
nucleus pulposus at C5-6. The C5 nerve root controls the elbow flexors, shoulder abductors,
and external rotators. The C7 nerve root controls the elbow extensors, wrist pronators, and
the triceps reflex. PREFERRED RESPONSE: 4

7. A 42-year-old woman underwent an instrumented posterior spinal fusion at


L3-S1 with transforaminal lumbar interbody fusion. She had an excellent clinical
result with complete resolution of leg pain. Three months later she now reports
increasing back pain and weakness in her legs. Examination reveals weakness in the
quadriceps and tibialis anterior. Radiographs show no interval changes in the
position of the hardware. MRI scans are shown in Figures 2a through 2c. What is the
next most appropriate step in management?

1- Observation
2- Oral antibiotics only
3- IV antibiotics only
4- Irrigation and debridement of the surgical site
5- Irrigation and debridement of the surgical site with hardware removal

DISCUSSION: The MRI scans reveal a postoperative infection. Observation and antibiotics
are not appropriate choices. There is a large fluid collection and this requires decompression
5
AAOS 2009 Spine Self-Assessment Examination
because the patient has neurologic changes. There is considerable debate regarding the
removal of hardware. Many contend that biofilm on the implants can harbor the infection.
However, these complications usually can be treated with serial irrigations, debridements,
and IV antibiotics. The incidence of infection has been widely studied with varying rates in
fusions with instrumentation. Rates appear to be increased with instrumentation, yet these
infections usually can be managed without hardware removal. PREFERRED RESPONSE: 4

8. What is the primary reason for including the ilium in the distal fixation of long
instrumentation constructs in adult scoliosis?

1- Better coronal balance


2- Better pelvic balance
3- Reduced fretting and corrosion
4- Improved curve correction
5- Improved fusion success

DISCUSSION: Studies have shown that when compared with fixation to the sacrum alone,
the success rate of fusion across the lumbosacral junction increases when both the sacrum
and ilium are included in the posterolateral construct. Curve correction, coronal balance, and
pelvic balance are all attended to within the thoracolumbar spine and are not directly related
to the pelvic fixation. Fretting and corrosion are a byproduct of metal-to-metal connections.
PREFERRED RESPONSE: 5

9. A 60-year-old man is evaluated in the ICU after a rollover motor vehicle


accident 3 days ago. He has multiple upper and lower extremity trauma and was
found unresponsive at the accident scene. Surgery is planned for the extremity
trauma once the patient is medically stable. He remains intubated and the cervical
spine is immobilized in a semi-rigid collar. Examination reveals mild erythema in the
posterior occipital cervical region. Initial AP and lateral radiographs of the cervical
spine have not revealed any obvious fracture. What is the most appropriate
treatment option at this time?

1- Continued semi-rigid immobilization until the extremity surgeries are


completed
2- Halo skeletal fixation prior to the extremity surgery
3- Definitive clearance of the cervical spine with CT and/or MRI
4- Removal of the semi-rigid collar and physical examination when the
patient is responsive
5- Soft collar immobilization and local wound care

DISCUSSION: Ackland and associates demonstrated that the failure to achieve early spinal
clearance in an unconscious blunt trauma patient predisposed the patient to increased
morbidity secondary to the prolonged used of cervical immobilization. They demonstrated
that the four significant predictors of collar-related ulcers were ICU admission, mechanical
ventilation, the necessity for cervical MRI, and the time to cervical spine clearance and collar
removal. The risk of pressure-related ulceration increased by 66% for every 1-day increase

6
AAOS 2009 Spine Self-Assessment Examination
in Philadelphia collar time and this highlights the need for definitive C-spine clearance.
PREFERRED RESPONSE: 3

10. A 46-year-old woman who was involved in a motor vehicle accident reports a
4-month history of right-sided lower back pain and pain radiating into the right thigh.
The patient underwent an extensive 3-month course of physical therapy and now is
dependent on narcotic medication for pain control. Epidural injection therapy has
failed to improve her symptoms. Examination is significant for weakness of hip
flexion in the seated position and for decreased sensation to light touch in the
medial anterior thigh region. Straight leg raise is negative, but the femoral stretch
test reproduces anterior thigh pain. A CT myelogram image, at L3-L4, is shown in
Figure 3. What is the most appropriate management at this time?

1-Repeat epidural steroid injections


2-Wide lumbar laminectomy
3-Microdiskectomy from either a midline approach or
far lateral approach
4-Referral to pain management
5-Minimally invasive posterior lumbar interbody fusion

DISCUSSION: The CT scan reveals a right-sided lateral disk


protrusion at L3-4 that has been symptomatic for more than 4
months despite appropriate nonsurgical management.
Relative surgical indications include persistent radiculopathy
despite an adequate trial of nonsurgical management, recurrent episodes of sciatica,
persistent motor deficit with tension signs and pain, and pseudoclaudication caused by
underlying stenosis. Whereas studies have shown improvement in patients with sciatica from
a lumbar disk herniation treated either nonsurgically or surgically, those undergoing surgical
treatment had an overall greater improvement of symptoms. PREFERRED RESPONSE: 3

11. A 73-year-old woman reports a 4-month history of severe left-sided posterior


buttock pain and left leg pain. The leg pain radiates into the left lateral thigh and
posterior calf with cramping. Examination reveals mild difficulty with a single-leg toe
raise on the left side and a diminished ankle reflex. There is also a significant straight
leg raise test at 45 degrees which exacerbates symptoms. An MRI scan is shown in
Figure 4. What is the most appropriate treatment at this time?

1-Lumbar laminectomy with synovial cyst excision


2- Repeat epidural steroid injection
3- Microdiskectomy at L4-5
4-Nonsteroidal medication and outpatient physical
therapy
5- Left-sided facet blocks at L4-5 and L5-S1

7
AAOS 2009 Spine Self-Assessment Examination
DISCUSSION: Lumbar spinal stenosis with lumbar radiculopathy can be commonly caused
by a synovial cyst arising from the facet joints. Lyons and associates reported on the surgical
treatment of synovial cysts in 194 patients. Of the 147 with follow-up data, 91% reported
good pain relief and 82% had improvement of their motor deficits. Epstein reported a 58% to
63% incidence of good/excellent results and a 38 to 42 point improvement on the SF-36
Physical Function Scale. It was also suggested that since the presence of a synovial cyst
indicates facet pathology, possible fusion should be considered in these patients, especially
those with underlying spondylolisthesis.

PREFERRED RESPONSE: 1

12. Osteoporotic vertebral compression fractures are associated with

1- neurologic deterioration in 33% of patients.


2- osteomalacia in 50% of patients.
3- a further fracture risk rate of 20%.
4- chronic pain in 75% of patients.
5- a 2-year mortality rate that is less than that associated with hip
fractures.

DISCUSSION: Osteoporotic vertebral compression fractures are associated with neurologic


complications in less than 1% of patients. After the initial fracture however, patients have a
20% risk of further fractures. The mortality rate of patients with vertebral fractures exceeds
that of patients with hip fractures when they are followed beyond 6 months.

PREFERRED RESPONSE: 3

13. When compared to smokers who do not quit, an improvement in the rate of
lumbar fusion is seen in patients who cease smoking for at least how many months
postoperatively?

1- 1 month
2- 2 months
3- 4 months
4- 6 months
5- 12 months

DISCUSSION: The effects of cigarette smoking and smoking cessation on spinal fusion have
been studied extensively. Although permanent smoking cessation is ideal, significant
improvements in fusion rates are seen in patients who avoid smoking for greater than 6
months postoperatively.

PREFERRED RESPONSE: 4

8
AAOS 2009 Spine Self-Assessment Examination
14. A 19-year-old woman reports persistent neck pain for 2 years. Pain is relieved
with aspirin. A bone scan shows intense uptake in the superior, posterior portion of
the C3 vertebral body. A sagittal CT reconstruction is shown in Figure 5. Treatment
should consist of

1- radiation therapy.
2- en bloc excision.
3- posterior fusion at C2-C3 with instrumentation.
4- CT-guided aspiration followed by IV antibiotics.
5- radiofrequency ablation.

DISCUSSION: The CT scan shows an osteoblastic nidus


pathognomic for an osteoid osteoma. Surgical treatment
should include an en bloc excision of the lesion. Surgical
treatment is not mandatory because the lesion often becomes
asymptomatic over time. This lesion is not amenable to
radiofrequency ablation due to its proximity to the spinal cord.
A complete corpectomy is not necessary to adequately resect
the lesion, as only the nidus needs to be removed. Radiation
therapy and antibiotics are not appropriate treatments for an osteoid osteoma. Posterior C2-
C3 fusion will not address the pathology. PREFERRED RESPONSE: 2

15. A 56-year-old man with a history of chronic lower back pain from lumbar
spondylosis reports a 2-day history of acute incapacitating back pain. He denies any
history of acute trauma, although he reports the pain starting after a coughing spell.
He also reports difficulty urinating and some fecal incontinence. Examination reveals
generalized lower extremity weakness, saddle paresthesia, hyporeflexia in the lower
extremities, and loss of rectal tone. What is the most appropriate management at
this time?

1- Immediate MRI of the lumbar spine


2- General reassurance, anti-inflammatory drugs, and an early home
exercise program
3- Immediate radiographs of the lumbar spine and pain medications with 2
days of bed rest if the radiographs are normal
4- Office caudal epidural steroid injection with follow-up in 1 week
5- Outpatient MRI of the lumbar spine with follow-up in 1 week for test
results

DISCUSSION: Cauda equina syndrome is a medical emergency that must be quickly


diagnosed and treated to avoid long-term complications. Cauda equina syndrome typically
presents with low back pain, unilateral or usually bilateral sciatica, saddle sensory
disturbances, bladder and bowel dysfunction, and variable lower extremity motor and sensory
loss. Although a number of pathologies can cause cauda equina syndrome, in a patient with
a history of chronic back pain, disk pathology is the most common cause of acute onset
9
AAOS 2009 Spine Self-Assessment Examination
cauda equina syndrome. Whereas radiographs may be useful in a traumatic onset of
symptoms, MRI is the most appropriate study. Cauda equina syndrome should be evaluated
on an emergent basis and admission for work-up is appropriate. PREFERRED RESPON: 1

16. A 55-year-old woman with a long history of low back and left lower extremity
pain has failed to respond to exhaustive nonsurgical management. MRI scans show
bulging and degeneration at L3-4 and L4-5 as well as a normal disk at L2-3 and L5-S1.
She undergoes provocative lumbar diskography at L3-4, L4-5, and L5-S1. Post-
diskography axial CT images of L3-4 and L4-5 are shown in Figures 6a and 6b,
respectively. The injections at L3-4 and L4-5 produce no pain. The injection at L5-S1
produces 10/10 concordant back pain with radiation to the lower extremity. What is
the most appropriate recommendation at this time?

1- Consider fusion surgery


2- Intradiskal ozone therapy
3- Lumbar laminectomy
4- Vertebral augmentation
5- Cognitive intervention and exercise

DISCUSSION: The results of this patient’s lumbar diskography are equivocal at best. The
two disks most likely to be her pain generators, based on their MRI appearance, produced
10/10 pain, however it was nonconcordant and did not reproduce any of her typical left-sided
radicular symptoms. The only disk that produced concordant back pain was the normal disk
at the L5-S1 level and it reproduced radicular symptoms on the side opposite of her typical
pain. Based on these findings, it would be difficult to select a level or levels to include in a
lumbar fusion. As such, continued nonsurgical management is the safest treatment option at
the current time. Brox and associates reported on a randomized clinical trial comparing
lumbar fusion to cognitive intervention and exercise and found similar results in both groups,
with significantly less risk in the latter. PREFERRED RESPONSE: 5
10
AAOS 2009 Spine Self-Assessment Examination
17. A 36-year-old woman is brought to the emergency department intubated and
sedated following a motor vehicle accident. She is moving her upper and lower
extremities spontaneously. She cannot follow commands. CT scans are shown in
Figures 7a through 7c. The initial survey does not reveal any other injuries. Initial
management of the cervical injury should consist of immediate

1- immobilization with a halo ring and vest with reduction when medically stable.
2- closed traction reduction using Gardner-Wells tongs.
3- posterior open reduction, stabilization, and fusion.
4- cervical MRI followed by reduction.
5- anterior open reduction, stabilization, and fusion.

DISCUSSION: The patient has a bilateral facet dislocation of C6-C7 with preservation of at
least some neurologic function. Urgent reduction is necessary. However, because she is
sedated and unable to follow commands, an MRI scan is necessary before any closed or
open posterior reduction to look for an associated disk herniation. If a disk herniation is
present, it must be removed prior to any reduction maneuver to prevent iatrogenic neurologic
injury. It is very unlikely that this injury can be reduced with an open anterior procedure
alone.PREFERRED RESPONSE: 4

18. A 51-year-old woman with no preoperative neurologic deficit is undergoing


elective anterior cervical diskectomy and fusion (ACDF) with plating and fusion for a
C5-6 disk herniation with right-sided neck pain. Thirty minutes into the surgery the
neurophysiologic monitoring shows a rapid drop and then loss of amplitude in the
right cortical somatosensory-evoked potential waveform. All other waveforms
remained normal and unchanged, including right-sided cervical (subcortical) and
peripheral (Erb’s point), and those from the left-sided upper extremity and both
lower extremities. What is the most likely cause of the change?

1- Electrode placement
2- Stimulation failure
3- Anesthetic effect
4- Cord ischemia from retraction
5- Cerebral ischemia from retraction
11
AAOS 2009 Spine Self-Assessment Examination
DISCUSSION: The change noted is focal and confined to the cortex, sparing the opposite
side, both lower extremities, and the subcortical waveforms, making all the choices unlikely
with the exception of carotid compression with focal cortical ischemia. This may be
associated with poor collateral flow from the opposite hemisphere due to an incomplete circle
of Willis.PREFERRED RESPONSE: 5

19. A 68-year-old woman undergoes a complicated four-level anterior cervical


diskectomy and fusion at C3-7 with iliac crest bone graft and instrumentation for
multilevel cervical stenosis. Surgical time was approximately 6 hours and estimated
blood loss was 800 mL. Neuromonitoring was stable throughout the procedure. The
patient’s history is significant for smoking. The most immediate appropriate
postoperative management for this patient should include

1- normal postoperative orders with frequent neurologic evaluations for


the first 24 hours.
2- administration of IV steroids and placement of a soft cervical collar for
24 hours.
3- placement of both deep and superficial surgical drains prior to wound
closure.
4- administration of IV mannitol and placement of a soft collar.
5- maintaining intubation for up to 24 to 48 hours.

DISCUSSION: Airway complications after anterior cervical surgery can be a catastrophic


event necessitating emergent intubation for airway protection. Multilevel surgeries requiring
long intubation and prolonged soft-tissue retraction as well as preexisting comorbidities may
predispose a patient to postoperative airway complications. Sagi and associates reported
that surgical times greater than 5 hours, blood loss greater than 300 mL, and multilevel
surgery at or above C3-4 are risk factors for airway complications. In surgical procedures
with the aforementioned factors, serious consideration should be given to elective intubation
for 1 to 3 days to avoid urgent reintubation. PREFERRED RESPONSE: 5

20. A 22-year-old woman reports a 4-year history of worsening low back and left
lower extremity pain following a motor vehicle accident. Management consisting of
physical therapy, chiropractic manipulation, and interventional pain management,
including sacroiliac joint injections and epidural steroid injections, has failed to
provide relief. A sagittal T2-weighted MRI scan is shown in Figure 8. No nerve root
compression is seen on axial images. She is currently working and lives with her
fiancé. She smokes half a pack of cigarettes per day and reports depression on her
health history. She is being maintained on narcotic analgesics and is having
increasing difficulty performing her activities of daily living secondary to pain. What
is the most appropriate management at this time?

12
AAOS 2009 Spine Self-Assessment Examination
1-Provocative lumbar diskography
2-Laboratory studies, including a complete blood cell (CBC)
count, erythrocyte sedimentation rate (ESR), and urinalysis
3-Cognitive intervention, exercise, and smoking cessation
4-Bilateral lower extremity electromyography and nerve
conduction velocity studies
5-Lumbar myelogram with a postmyelography CT scan of
the lumbar spine

DISCUSSION: The MRI scan reveals a rudimentary disk at the


L5-S1 level, suggesting transitional anatomy. There is a
posterior disk bulge at L3-4. At L4-5, there is disk desiccation
and loss of disk height, with a posterior disk bulge and a high
intensity zone in the posterior annulus, suggesting an annular
tear. While these and similar radiographic findings have been
associated with the severity of a patient’s pain, they are also
commonly found in cross-sectional studies of asymptomatic
subjects. Carragee and associates found 59% of symptomatic patients undergoing
diskography have high intensity zones as compared to 25% of asymptomatic subjects of a
similar patient profile. Diskographic injections provoked pain in disks with high intensity
zones approximately 70% of the time whether the individual was previously symptomatic or
not. This patient’s non-specific pain pattern does not require further work-up as she is not a
surgical candidate. PREFERRED RESPONSE: 3

21. A 42-year-old man with a history of renal cell carcinoma has progressive
weakness in the lower extremities for the past 3 weeks. The patient desires
intervention. A sagittal T2-weighted MRI scan is shown in Figure 9a, and a sagittal
contrast enhanced T1-weighted MRI scan is shown in Figure 9b. He currently
ambulates minimal distances with a walker. His life expectancy is 8 months.
Treatment of the spine lesion should consist of

1- radiation therapy.
2- posterior laminectomy.
3- anterior corpectomy and reconstruction.
4- posterior laminectomy and fusion.
5- kyphoplasty.
13
AAOS 2009 Spine Self-Assessment Examination
DISCUSSION: The MRI scans show a metastatic lesion in two contiguous vertebral bodies in
the lower thoracic spine. Posterior laminectomy is not indicated because this does not
adequately decompress the neural elements and will lead to progressive kyphosis. A
posterior fusion may prevent progressive kyphosis but will not decompress the spinal cord.
Renal cell carcinoma is not radiosensitive; therefore, radiation therapy would not be helpful in
relieving neurologic compression. The lesion should be treated by an anterior corpectomy
and reconstruction. This will allow for complete decompression as well as reconstruction of
the anterior column. Kyphoplasty is not indicated in a lesion with disruption of the posterior
cortex and neurologic impairment. PREFERRED RESPONSE: 3

22. A 40-year-old man has intractable pain following 2 years of nonsurgical


management for high-grade spondylolisthesis. What is the best surgical option?

1- Posterolateral fusion
2- Posterolateral fusion with instrumentation
3- Circumferential fusion
4- Transforaminal lumbar interbody fusion
5- Anterior lumbar interbody fusion

DISCUSSION: Circumferential fusion is the preferred choice for patients undergoing revision
surgery following failed posterolateral fusions for isthmic spondylolisthesis as well as for
those patients having primary surgery for high-grade isthmic spondylolisthesis.PRE RESP: 3

23. An adult patient with a grade I isthmic spondylolisthesis at L5-S1 is most likely
to have weakness of the

1- anterior tibialis.
2- quadriceps.
3- gastrocsoleus.
4- extensor hallucis longus.
5- iliopsoas.

DISCUSSION: Adult patients with isthmic spondylolisthesis most commonly have neurologic
symptoms due to foraminal stenosis at the level of the spondylolisthesis. In this scenario, the
patient is most likely to have weakness of the L5 myotome, which would cause weakness of
the extensor hallucis longus. PREFERRED RESPONSE: 4

24. When performing a long fusion to the sacrum in an osteopenic patient in


whom optimal sagittal balance is restored, which of the following is a benefit of
extending the distal fixation to the pelvis, rather than the sacrum alone?

1- Decreased risk of sacral fractures


2- Decreased risk of proximal functional kyphosis
3- Easier contouring of the instrumentation
4- Reduced risk of late pubic ramus fractures
5- Improved coronal plane correction
14
AAOS 2009 Spine Self-Assessment Examination
DISCUSSION: In osteopenic individuals, even those with excellent obtained or maintained
balance, long instrumented fusions to the sacrum impart a high degree of strain, and the
sacrum may fail in a transverse fracture or fracture-dislocation pattern. The risk of proximal
functional kyphosis is unrelated to distal fixation as are coronal plane correction and rod
contouring. Pubic ramus fractures have been shown to be associated with both fixation to
the sacrum alone as well as to the ilium. PREFERRED RESPONSE: 1

25. Which of the following statements describing chordomas is false?

1- Treatment consisting of complete surgical resection with clean margins


offers the best survival.
2- They occur in the clivus and sacrum and occur only 15% of the time in
the rest of the spine.
3- They are locally aggressive and invasive tumors.
4- They are highly radiosensitive.
5- They have the ability to become malignant.

DISCUSSION: Casali and associates provided a recent review of the treatment options for
chordomas. These tumors are not radiosensitive; however, modern intensity modulated
radiosurgery techniques may be of value. The combination of surgery and radiotherapy
compared to surgery alone results in the same disease-free survival time. Complete surgical
resection of the chondroma with clean margins offers the best survival; however, its location
may make total removal impossible. Thus subtotal resection followed by radiotherapy results
in better survival despite the tumor’s lack of radiosensitivity. PREFERRED RESPONSE: 4

26. A previously healthy 35-year-old man was involved in a rollover motor vehicle
accident 2 days ago. He was placed in a semi-rigid cervical orthosis. He now reports
mostly axial neck pain with attempted range of motion. Examination reveals the
mechanical neck pain but no obvious neurologic deficits. AP, flexion, and extension
radiographs are shown in Figures 10a through 10c, and sagittal and coronal CT scans
are shown in Figures 10d and 10e. What is the most appropriate management at this
time?

15
AAOS 2009 Spine Self-Assessment Examination

1- Continued immobilization in a semi-rigid cervical orthosis for 6 to 8 weeks


2- Posterior occipital-cervical fusion with iliac crest bone graft
3- Open reduction and internal fixation of the odontoid process with an anterior
odontoid screw
4- Resection of the odontoid process through a transoral approach
5- Reduction with Gardner-Wells tong traction and 6 weeks of skeletal traction

DISCUSSION: Odontoid fractures can be classified based on the anatomic position of the
fracture within the dens itself. Type I is an oblique fracture through the upper part of the
odontoid process. Type II is a fracture that occurs at the base of the odontoid as it attaches
to the body of C2; type III occurs when the fracture line extends through the body of the axis.
Type 1 fractures typically can be treated nonsurgically with 6 to 8 weeks of immobilization
with a semi-rigid cervical orthosis. Nondisplaced, deep type III fractures generally are treated
with skeletal halo fixation. Deep, displaced, and angled type III fractures can be treated with
closed reduction and skeletal halo fixation. Shallow type III fractures are sometimes
amenable to anterior odontoid screw fixation. Type II fractures can be managed
nonsurgically or surgically. Treatment options include halo immobilization, internal fixation
(odontoid screw fixation), and posterior atlantoaxial arthrodesis. Management with the halo
vest usually is considered if the initial dens displacement is less than 6 mm, the reduction is
performed within 1 week of the injury and is able to be maintained, and the patient is younger
than age 60 years. Halo vest immobilization can lead to a healing rate of more than 90%.
Posterior surgical fusion techniques provide high fusion success rates but do so at the
expense of cervical rotation. Up to 50% of rotation is lost with these techniques. Anterior
odontoid single screw fixation is often tolerated better than skeletal halo fixation and also is
noted to preserve the normal rotation at C1/C2. Studies have shown less of a malunion and
nonunion rate in the treatment of type II odontoid fractures with anterior odontoid screw
fixation. Osteoporosis, short neck and barrel-chested anatomy, and fractures that are more
than 4 weeks old preclude anterior odontoid fixation.

PREFERRED RESPONSE: 3

16
AAOS 2009 Spine Self-Assessment Examination
27. Which of the following palpable bony landmarks is correctly matched with its
corresponding vertebral level?

1- Angle of the mandible and the C2-C3 interspace


2- Hyoid bone and C6
3- Carotid tubercle and C6
4- Superior portion of the thyroid cartilage and the C3 vertebral body
5- Cricoid cartilage and C7-T1

DISCUSSION: The carotid tubercle is usually located at the level of C6. The angle of the
mandible is at C1-C2; the hyoid is at C4; the superior portion of the thyroid cartilage is C4-C5;
and the cricoid cartilage is at C6. PREFERRED RESPONSE: 3

28. What root is most commonly involved with a segmental root level palsy after
laminoplasty?

1- C3
2- C4
3- C5
4- C6
5- C7

DISCUSSION: The postoperative incidence of C5 root palsy after laminoplasty ranges from
5% to 12%. Other roots also may be affected. The palsies tend to be motor dominant,
although sensory dysfunction and radicular pain are also possible. The palsy may arise
during the immediate postoperative period or up to 20 days later. C5 may be preferentially
involved because it is at the apex of the cervical lordosis. Recovery usually occurs over
weeks to months.PREFERRED RESPONSE: 3

29. Up to what time frame are the risks minimized in anterior revision disk
replacement surgery?

1- 3 days
2- 1 week
3- 10 days
4- 2 weeks
5- 6 weeks

DISCUSSION: Revision anterior exposure within 2 weeks of total disk replacement incurs
relatively little additional morbidity because adhesion formation is minimal. Surgeons should
have a low threshold for revising implants that are clearly dangerously malpositioned or show
early migration within this 2-week window. Beyond this time period, a revision strategy must
be individualized to the particular clinical situation. A posterior fusion with instrumentation
with or without a laminectomy is currently the most effective salvage procedure.
PREFERRED RESPONSE: 4

17
AAOS 2009 Spine Self-Assessment Examination
30. Which of the following best describes the use of epidural morphine and steroid
paste after laminectomy?

1- Associated with an 11% rate of postoperative surgical site complications


2- Associated with a less than 1% rate of surgical site infections
3- Associated with a decreased rate of postoperative urinary retention
4- Considered the standard for outpatient microdiskectomy
5- Should only be used in the absence of radiculopathy

DISCUSSION: Kramer and associates conducted a retrospective review during an “epidemic”


period to identify the risk factors associated with a sudden increase in the rate of surgical site
infections. They found in a multivariate analysis that the use of morphine nerve paste
resulted in a 7.6-fold increase in postoperative surgical wound debridement, and an 11% rate
of surgical site complications. PREFERRED RESPONSE: 1

31. Figures 11a and 11b show the T2-weighted MRI scans of the lumbar spine of a
53-year-old woman who has low back and right lower extremity pain. What
structure is the arrow pointing to in Figure 11a?

1- Ligamentum flavum
2- Lumbar synovial cyst
3- Tarlov cyst
4- Pseudomeningocele
5- Herniated nucleus pulposus

DISCUSSION: The arrow is pointing to a cystic-appearing structure with high signal intensity
on T2-weighted image sequencing. It appears to be contiguous with the hypertrophied right
facet joint, which appears to also have high signal intensity. The mass significantly narrows
the right lateral recess. The high signal intensity suggests that this is a fluid-filled mass. In
addition, the facet joints are degenerative and there is a very mild degree of anterolisthesis

18
AAOS 2009 Spine Self-Assessment Examination
on the sagittal image. These findings make a lumbar synovial cyst the most likely diagnosis.
Most lumbar juxtafacet cysts are observed at the L4-5 level, extradurally and adjacent to the
degenerative facet joint. They may contain synovial fluid and/or extruded synovium.
Presentation is indistinguishable from that of a herniated disk. The etiology of spinal cysts
remains unclear, but there appears to be a strong association between their formation and
worsening spinal instability. They occasionally regress spontaneously and may respond to
aspiration and injection of corticosteroids, though there is a high recurrence rate with
nonsurgical management. Synovial cysts resistant to nonsurgical management should be
treated surgically. If the patient’s symptoms can be attributable to radicular findings, a
microsurgical decompression that limits further destabilization should suffice. However, if
there is significant low back pain attributable to spinal instability, decompression and fusion
remains an appropriate option.

PREFERRED RESPONSE: 2

32. A 38-year-old man reports a 2-week history of acute lower back pain with
radiation into the left lower extremity. There is no history of trauma and no
systemic signs are noted. Examination reveals a positive straight leg test at 35
degrees on the left side and a contralateral straight leg raise on the right side. Motor
testing demonstrates mild weakness of the gluteus medius and weakness of the
extensor hallucis longus of 3+/5. Sensory examination demonstrates decreased
sensation along the lateral aspect of the calf and top of the foot. Knee and ankle
reflexes are intact and symmetrical. Radiographs demonstrate no obvious
abnormality. MRI scans show a posterolateral disk hernation. The diagnosis at this
time is consistent with a herniated nucleus pulposus at

1- L1-2.
2- L2-3.
3- L3-4.
4- L4-5.
5- L5-S1.

DISCUSSION: The patient’s history and physical examination findings are consistent with a
lumbar disk herniation at the L4-5 level. Weakness of the extensor hallucis longus and
gluteus medius are consistent with an L5 lumbar radiculopathy. Nerve root tension signs are
also consistent with sciatica from a lumbar disk herniation. The MRI scans confirm a
posterolateral disk herniation at L4-5, which typically affects the exiting L5 nerve root.

PREFERRED RESPONSE: 4

19
AAOS 2009 Spine Self-Assessment Examination
33. A 42-year-old woman is brought to the emergency department following a
motor vehicle accident. She has sustained multiple injuries, and she is intubated and
pharmacologically paralyzed. Sagittal cervical CT scans through the right cervical
facets, the left cervical facets, and the midline are shown in Figures 12a through 12c,
respectively. Definitive management of her cervical injury should consist of

1- anterior diskectomy and fusion at C4-C5.


2- immobilization in a Philadelphia collar and voluntary flexion and
extension radiographs when awake.
3- occipital-cervical fusion with instrumentation.
4- halo immobilization for 12 weeks.
5- left C6 superior facetectomy and posterior fusion at C6-C7 with
instrumentation.

DISCUSSION: The CT scans reveal an occipital-cervical dissociation with subluxation of the


occipitocervical joints bilaterally. Definitive management should consist of an occipital-
cervical fusion with instrumentation. Immobilization in a Philadelphia collar is inadequate for
this highly unstable injury, and halo immobilization, while affording adequate temporary
immobilization, is not appropriate definitive management for this ligamentous injury. The
patient does not have an injury at C4-C5 or C6-C7. PREFERRED RESPONSE: 3

34. A 32-year-old motorcycle rider is involved in a motor vehicle accident and


radiographs show a burst fracture at L2 with 20 degrees of kyphosis. The neurologic
examination is consistent with unilateral motor and sensory involvement of the L5,
S1, S2, S3, and S4 nerve roots. He has no other injuries. CT demonstrates 20%
anterior canal compromise with displaced laminar fractures at the level of injury.
What is the best option for management of this patient?
1- Bed rest for 6 weeks, followed by mobilization in a thoracolumbosacral
orthosis until the fracture has healed
2- Anterior corpectomy with strut grafting and placement of an anterior plate
spanning L1 to L3
3- Anterior corpectomy with strut grafting, followed by posterior spinal fusion
and instrumentation
4- Posterior spinal fusion and instrumentation from T11 to L4
5-L2 laminectomy and posterior spinal fusion and instrumentation from T11 to L4
20
AAOS 2009 Spine Self-Assessment Examination
DISCUSSION: The patient has a burst fracture with probable unilateral entrapment of the
cauda equina within the elements of the fractured lamina. A dural tear is likely in this
scenario as well. It is recommended that this type of burst fracture be treated surgically with
laminectomy, freeing of the entrapped nerve roots, and dural repair followed by stabilization
of the fracture by either a posterior or combined approach. The degree of kyphosis and the
extent of anterior canal compromise does not warrant corpectomy in this patient. Therefore,
after completing the laminectomy and dural repair, posterior fusion and instrumentation
should be sufficient to stabilize the fracture. PREFERRED RESPONSE: 5

35. A patient who underwent a L4-L5 hemilaminotomy and partial diskectomy for
radiculopathy 8 weeks ago now reports increasing low back pain without neurologic
symptoms. A sagittal T2-weighted MRI scan is shown in Figure 13a, and a contrast
enhanced T1-weighted MRI scan is shown in Figure 13b. What is the most
appropriate management for the patient’s symptoms?

1- Physical therapy
2- CT-guided needle biopsy and IV antibiotics
3- Revision laminotomy and diskectomy
4- L4-L5 anterior debridement and fusion
5- Open repair of the L4-L5 pseudomeningocele

DISCUSSION: The MRI scans show Modic changes in the L4-L5 vertebral bodies due to
spondylosis. There is no increased fluid signal or enhancement in the L4-L5 disk to suggest
infection or any other pathologic process. Therefore, the patient’s pain should be treated with
a course of physical therapy and rehabilitation. There is no infection; therefore, IV antibiotics
and debridement are not indicated. Similarly, a pseudomeningocele is not present. A
revision diskectomy is useful for recurrent radiculopathy but would not be helpful for
degenerative low back pain. PREFERRED RESPONSE: 1

36. What is the heaviest weight that can be safely applied to the adult cervical
spine via Gardner-Wells tong traction?
1- 40 pounds
2- 50 pounds
3- 75 pounds
4- 100 pounds
5- Greater than 100 pounds
21
AAOS 2009 Spine Self-Assessment Examination
DISCUSSION: Cotler and associates reported on the use of awake skeletal traction to reduce
facet fracture-dislocations in 24 patients. Seventeen patients required more than 50 pounds
of traction (the “traditional” limit) to achieve reduction. More than 100 pounds of traction was
safely used in one-third of the patients in this study. A cadaver study has supported the safe
use of traction with weights in excess of 100 pounds. PREFERRED RESPONSE: 5

37. A 68-year-old man reports a 4-week history of progressive left-sided lower


back and hip pain. The pain is in the posterior buttock region with radiation to the
groin and to the left anterior knee region. The pain is aggravated with walking and
improves with rest. There is no history of previous trauma. Radiographs are seen in
Figures 14a and 14b, and MRI scans are seen in Figures 14c through 14e. What is the
most appropriate treatment option at this time?

1- Epidural steroid injection at L4-5


2- Outpatient physical therapy for the lower back
3- Non-weight-bearing of the left lower extremity
4- Home exercise program, analgesics, and limited use of muscle relaxants
5- Cortisone injection of the left greater trochanter region
22
AAOS 2009 Spine Self-Assessment Examination
DISCUSSION: Although the imaging reveals generalized lumbar spondylosis and stenosis, in
particular at L4-5, the MRI scan of the left hip clearly reveals a stress fracture of the femoral
neck. Therefore, the treatment of choice is non-weight-bearing of the left lower extremity.
During the evaluation of acute back pain, clinicians must include other possibilities within the
differential diagnosis that may mimic mechanical axial back pain; thus, potential
complications from a missed diagnosis can be avoided. PREFERRED RESPONSE: 3

38. A 79-year-old woman reports a history of left leg pain with walking. Her pain
is exacerbated with walking and stair climbing, and her symptoms are improved by
standing after she stops walking. Lumbar flexion does not provide any significant
improvement of the symptoms and sitting does not significantly change symptoms.
Her leg pain is worse at night and she obtains relief by hanging her leg over the side
of the bed. The neurologic examination is essentially normal. Examination of the
lower extremities demonstrates mild early trophic changes, and her pulses distally
are palpable but are diminished bilaterally. Radiographs are shown in Figures 15a
and 15b. What is the next most appropriate step in management?

1- Decompression and posterior fusion at L4-L5


2- Epidural steroid injection at L4-5
3- Nonsteroidal medications and physical therapy for 6 weeks
4- Measurement of the ankle-brachial index
5- CT myelogram

DISCUSSION: The patient has symptoms that are more consistent with vascular claudication
than with the pseudoclaudication anticipated from lumbar spinal stenosis. Therefore, the
patient is a candidate for further vascular work-up. The radiographs reveal early spinal
stenosis and spondylolisthesis at L4-5 but also show significant calcification of the iliac
arteries, suggestive of peripheral vascular disease. Vascular claudication is a manifestation
of peripheral vascular disease and presents with crampy leg pain that is exacerbated by
physical exertion. The pain is easily relieved by standing still or sitting. Unlike
pseudoclaudication, a forward-flexed posture and/or sitting does not improve the symptoms.
Night pain is common in vascular claudication due to the elevation of the extremities and
patients often report pain improvement by hanging their extremities in a dependent position.
23
AAOS 2009 Spine Self-Assessment Examination
In evaluation of a patient with suspected vascular claudication, the five “P’s” of vascular
insufficiency should be monitored, including pulselessness, paralysis, paresthesia, pallor, and
pain. While pain and paresthesias can be common in both vascular claudication and
pseudoclaudication, the presence of any of the remaining symptoms is suggestive of vascular
disease. PREFERRED RESPONSE: 4

39. Figure 16 shows the MRI scan of a 43-year-old man who has had worsening
low back pain for the past 4 months. What is the
most likely diagnosis?

1- Osteochondroma
2- Posttraumatic kyphosis
3- Staphylococcus aureus osteomyelitis
4- Ankylosing spondylitis
5- Tuberculosis

DISCUSSION: Tuberculosis of the spine is seen in 50% to


60% of skeletal disease and is most commonly found in the
lower thoracic or upper lumbar spine. Typically two or more
adjacent bodies are involved as seen in this MRI scan. The
disk space is narrowed but still relatively preserved as
opposed to pyogenic infections (black arrow). Epidural
extensions often spread from vertebrae to vertebrae (white arrow); however, the posterior
elements are not frequently involved (arrowhead). Tumors rarely spread to adjacent
vertebrae. The anterior and posterior spread of the infectious process rules out trauma.
PREFERRED RESPONSE: 5

40. In patients without spondylolisthesis or scoliosis undergoing laminectomy for


lumbar spinal stenosis, spinal fusion is generally recommended if

1- a dural tear is repaired.


2- more than one level requires decompression.
3- less than one half of each facet is removed bilaterally.
4- the pars interarticularis is fractured.
5- the patient is a smoker.

DISCUSSION: With the notable exception of fusion for degenerative spondylolisthesis and
scoliosis, there is a paucity of evidence on the indications for spinal fusion in patients
undergoing laminectomy for spinal stenosis. However, it is generally recommended that if
the spine is destabilized (for example by removal of one complete facet joint or by an
iatrogenic pars fracture), spinal fusion should be considered. Although fusion can be
considered for a very long laminectomy, a two-level laminectomy does not represent, by
itself, a clear indication for the addition of a spinal fusion. The repair of a dural tear and the
use of nicotine by the patient play no role in the determination of whether or not to add fusion
to a laminectomy procedure. PREFERRED RESPONSE: 4
24
AAOS 2009 Spine Self-Assessment Examination
41. An 18-year-old collegiate basketball player has had a 3-month history of
activity-related back pain. She describes isolated low back pain without radiation
that increases with training and playing basketball. Her pain resolves with rest.
Physical therapy for 6 weeks has failed to provide relief. An axial CT scan is shown in
Figure 17a, and Figures 17b and 17c show sagittal CT reconstructions through the
right and left lumbar facets, respectively. Further management should consist of
which of the following?

1- CT-guided needle biopsy followed by radiation therapy


2- L5-S1 fusion with instrumentation
3- L5-S1 hemilaminotomy and partial diskectomy
4- Activity restriction and bracing
5- L5-S1 total disk arthroplasty

DISCUSSION: The sagittal and axial CT scans show a bilateral spondylolysis at L5. The
defect is in the pars interarticularis on the right side but at the base of the pedicle on the left.
Having failed a trial of physical therapy with only a 3-month history of pain, the next most
appropriate step in management should consist of activity modification and bracing in an
antilordotic lumbosacral orthosis. Surgical intervention is reserved for patients who have
failed to respond to a trial of bracing and activity restriction. PREFERRED RESPONSE: 4

42. Radiating pain associated with a posterolateral thoracic disk herniation


typically follows what pattern?

1- Extending down the spine into the lumbosacral region


2- Down the inner aspect of either upper extremity
3- Cephalad up to the cervicothoracic junction
4- Around or through the chest to the anterior wall
5- Down the contralateral lower extremity

DISCUSSION: Although symptomatic thoracic disk herniations can affect more caudal
structures, even to the point of paralysis, the pattern of radiating pain has been described as
either following the dermatomal band around the chest or feeling to the patient as if the pain
passes straight anteriorly to the chest wall. PREFERRED RESPONSE: 4
25
AAOS 2009 Spine Self-Assessment Examination
43. A 53-year-old man reports a 5-week history of worsening low back pain
accompanied by bilateral knee and ankle pain and swelling. He also reports a lesser
degree of neck and left elbow pain. He denies any history of trauma or provocative
episodes. His medical history is significant for Reiter’s syndrome more than 25 years
ago, with no subsequent exacerbations. Furthermore, he has recently returned from
a vacation in Costa Rica and noted the development of infectious gastroenteritis with
diarrhea within 1 week of his return. This was treated with a 10-day course of oral
antibiotics and has since resolved. He denies any significant bowel or urinary
symptoms at this time. His neurologic examination is essentially within normal
limits, but is somewhat limited by his low back and leg pain. What further
investigation is most appropriate at this time?

1- Radiographs of the lumbar spine and bilateral knees and ankles


2- MRI of the lumbar spine with and without gadolinium contrast
3- Synovial fluid analysis of the involved joints for crystals and bacteria
4- Laboratory tests including a CBC count, erythrocyte sedimentation rate
(ESR), and C-reactive protein (CRP)
5- Laboratory tests including CBC count, rheumatoid factor (RF),
antinuclear antibodies (ANA), and human leukocyte antigen-B27 (HLA-
B27)

DISCUSSION: The patient has pain involving the cervical and lumbar spine as well as pain
and swelling in both the knees and ankles. As such, this can be classified as polyarticular
arthritis. The presence of multiple joint symptoms in the lower extremities, the absence of a
history of trauma, and the multiple joints involved direct attention away from the spine as the
etiology of this patient’s pain. Radiographs of the involved joints are not likely to yield much
useful information to assist with a diagnosis. Likewise, an MRI scan of the lumbar spine is
not likely to provide much information regarding the etiology of the patient’s condition. When
a rheumatologic illness is suspected, the selective use of confirmatory laboratory testing can
aid in arriving at a correct diagnosis. A presumed case of gout or chondrocalcinosis can be
confirmed by the presence of the appropriate crystals in a joint-fluid aspiration. Because of
the patient’s recent trip to Costa Rica and the subsequent gastroenteritis, a CBC count, ESR,
and CRP should be ordered to rule out infectious and inflammatory versus noninflammatory
conditions. Rheumatoid factor (RF) in general should only be ordered for patients with
polyarticular joint inflammation for more than 6 weeks. The presence of rheumatoid factor
does not indicate rheumatoid arthritis. Antinuclear antibodies (ANA) should be ordered when
a connective tissue disease such as systemic lupus erythematosus (SLE) is suspected on the
basis of specific history and physical examination findings, such as inflammatory arthritis.
Human leukocyte antigen-B27 (HLA-B27) should be ordered only when the patient’s history
is compatible with ankylosing spondylitis or Reiter’s syndrome and this patient had a history
of Reiter’s syndrome.

PREFERRED RESPONSE: 4

26
AAOS 2009 Spine Self-Assessment Examination
44. The 5-year outcome for patients with sciatica secondary to lumbar disk
herniation shows which of the following results?

1- Patients have the same likelihood of receiving disability whether treated with
or without surgery.
2- Sixty percent of surgically treated patients undergo at least one more
operation within 5 years.
3- Only 20% of patients treated with surgery report improved symptoms of back
and/or leg pain.
4- A smaller portion of surgical patients, compared to nonsurgically treated
patients, report improvement.
5- Fifty percent of patients treated nonsurgically seek surgery within 5 years.

DISCUSSION: Atlas and associates, in the Maine Lumbar Spine Study, reported that overall,
patients treated initially with surgery reported better outcomes. By 5 years, 19% of surgical
patients had undergone at least one additional lumbar spine operation, and 16% of
nonsurgical patients had opted for at least one lumbar spine operation. At the 5-year follow-
up, 70% of patients initially treated surgically reported improvement in their predominant
symptom (back or leg pain) versus 56% of those initially treated nonsurgically. They also
noted that there was no difference in the proportion of patients receiving disability
compensation at the 5-year follow-up. PREFERRED RESPONSE: 1

45. What is one of the principle concerns when a fracture such as the one seen in
Figure 18 is encountered?

1- Fractures of the lower extremities


2- Paroxysmal hypertension
3- Infection
4- Epidural hematoma
5- Gastrointestinal bleeding

DISCUSSION: The injury shown is a fracture-dislocation and it is


highly unstable. In addition to this concern, spinal epidural
hematomas have a much higher incidence in people with ankylosing spondylitis following
knee fracture. It is felt to be due to disrupted epidural veins, with hypervascular epidural soft
tissue in the setting of a rigid spinal canal. Patients with ankylosing spondylitis may have
other significant comorbidities, especially cardiac and pulmonary, and these should be
carefully assessed. PREFERRED RESPONSE: 4
27
AAOS 2009 Spine Self-Assessment Examination
46. Retrograde ejaculation is most commonly associated with what surgical
approach?

1- Anterior retroperitoneal approach to L5-S1


2- Anterior transperitoneal approach to L5-S1
3- Anterior retroperitoneal approach to L4-5
4- Minimally invasive lateral trans-psoas approach to L4-5
5- Open lateral approach to L4-5

DISCUSSION: Retrograde ejaculation is the sequela of an injury to the superior hypogastric


plexus. This structure needs protection, especially during anterior exposure of the
lumbosacral junction. Although the superior hypogastric plexus can be injured with anterior
or anterolateral spine surgery at any lumbar level, it is most at risk with anterior
transperitoneal approaches to the lumbosacral junction. To avoid this complication, the use
of monopolar electrocautery should be avoided during deep dissection in this region. The
ideal anterior exposure starts with blunt dissection just to the medial aspect of the left
common iliac vein sweeping the prevertebral tissues toward the patient’s right side.

PREFERRED RESPONSE: 2

47. What nerve is most likely to be injured during the anterior exposure of C2-3?

1- Facial
2- Superior laryngeal
3- Vagus
4- Hypoglossal
5- Phrenic

DISCUSSION: The hypoglossal nerve exits from the ansa cervicalis at approximately the C2-
3 level and can be injured during retraction up to the C2 level. The superior laryngeal nerve
lies at about C4-5. The facial nerve is much higher. The vagus nerve runs with the internal
jugular and carotid much more laterally. The phrenic nerve exits posteriorly.

PREFERRED RESPONSE: 4

28
AAOS 2009 Spine Self-Assessment Examination
48. A 24-year-old man sustains the injury shown in Figures 19a through 19e in a
paragliding accident. He is neurologically intact. He also sustained fractures of his
left femur and right distal radius. Which of the following represents the best option
for management of the spinal injury?

1- Bed rest for 6 weeks, followed by mobilization in a thoracolumbosacral


orthosis (TLSO) until the fracture has healed
2- Immediate mobilization in a TLSO, continuing until the fracture has healed
3- Anterior corpectomy with strut grafting and placement of anterior fixation
4- Anterior corpectomy and strut grafting followed by posterior spinal fusion and
instrumentation
5- Posterior spinal fusion and instrumentation

DISCUSSION: The injury pattern is that of a burst fracture at L1 contiguous with a


compression fracture at T12. There is associated kyphosis and slight spondylolisthesis of
T12 on L1. Treatment of this type of burst fracture in neurologically intact patients is
somewhat controversial, with at least one study demonstrating equal long-term results
comparing nonsurgical treatment to surgical treatment. In this study, however, body casts
were used initially in the nonsurgical group. Moreover, because this patient has multiple
29
AAOS 2009 Spine Self-Assessment Examination
fractures, spinal fracture stabilization should be considered to facilitate early mobilization.
Surgical stabilization and fusion via a posterior approach is the best treatment option in this
patient. Anterior decompression is not necessary since the patient is neurologically intact.
PREFERRED RESPONSE: 5

49. An 82-year-old man is seen in consultation after being admitted for a fall from
ground level. There was no loss of consciousness and the patient recalls striking his
head and sustaining a hyperextension-type injury to the cervical spine. Examination
reveals an 8-cm head laceration with only mild axial neck tenderness. He has
generalized weakness throughout the upper extremities and maintained motor
function of the lower extremities. There are no obvious sensory deficits, and the
bulbocavernous reflex and deep tendon reflexes are maintained. What is the most
appropriate diagnosis at this time?

1- Anterior cord syndrome


2- Central cord syndrome
3- Posterior cord syndrome
4- Brown-Séquard syndrome
5- Spinal shock

DISCUSSION: Incomplete cord syndromes have variable neurologic findings with partial loss
of sensory and/or motor function below the level of injury. Incomplete cord syndromes
include the anterior cord syndrome, the Brown-Séquard syndrome, central cord syndrome,
and posterior cord syndrome. Central cord syndrome is characterized with greater motor
weakness in the upper extremities than in the lower extremities. The pattern of motor
weakness shows greater distal involvement in the affected extremity than proximal muscle
weakness. Anterior cord syndrome involves a variable loss of motor function and pain and/or
temperature sensation, with preservation of proprioception. The Brown-Séquard syndrome
involves a relatively greater ipsilateral loss of proprioception and motor function, with
contralateral loss of pain and temperature sensation. Posterior cord syndrome is a rare injury
and is characterized by preservation of motor function, sense of pain, and light touch, with
loss of proprioception and temperature sensation below the level of the lesion. Spinal shock
is the period of time, usually 24 hours, after a spinal injury that is characterized by absent
reflexes, flaccidity, and loss of sensation below the level of the injury.

PREFERRED RESPONSE: 2

50. Kyphosis from a vertebral osteoporotic compression fracture often results in


progressive kyphosis due to

1- progressive increase in lumbar lordosis.


2- load transfer to the superior adjacent vertebra.
3- normalization of load transfer with working kyphosis.
4- reduced strain at the occipito-cervical junction.
5- reduced strain at the apex of the deformity.

30
AAOS 2009 Spine Self-Assessment Examination
DISCUSSION: Kayanja and associates, in a number of biomechanical studies, showed that
in a kyphotic spine the strain is located at the apex of the deformity, the force is transmitted to
the superior adjacent vertebrae, and that realignment and cement augmentation effectively
normalize the load transfer. PREFERRED RESPONSE: 2

51. A 58-year-old woman with rheumatoid arthritis has progressive neck pain,
upper extremity and lower extremity weakness, and difficulty with fine motor
movements. Examination reveals hyperreflexia with mild to moderate objective
weakness but the patient has no difficulty with ambulation for short distances. What
is the most important preoperative imaging finding that predicts full neurologic
recovery with surgical stabilization?

1- Basilar invagination of less than 1 cm


2- Anterior atlanto-dens interval of 4 mm
3- Posterior atlanto-dens interval of greater than 14 mm
4- Rotatory subluxation of less than 10 degrees
5- Subaxial subluxation of less than 3.5 mm

DISCUSSION: Boden and associates’ article presents compelling evidence that patients with
rheumatoid arthritis and neurologic deterioration in C1-2 instability are more likely to achieve
some improvement if the posterior atlanto-dens interval is greater than 10 mm on
preoperative studies. All the patients in their series who had neurologic deterioration and a
preoperative posterior atlanto-dens interval of greater than 14 mm achieved complete motor
recovery.PREFERRED RESPONSE: 3

52. Figures 20a through 20d show the radiographs and MRI scans of a 59-year-old
woman who has had symptoms consistent with progressive neurogenic claudication
and back pain for the past 9 months. In the last 6 months, nonsurgical management
consisting of nonsteroidal anti-inflammatory drugs, physical therapy, and a series of
epidural steroid injections have been used; however the injections, while beneficial,
have provided only temporary relief of her symptoms. What is the most appropriate
management at this time?

31
AAOS 2009 Spine Self-Assessment Examination
1- Repeat trial of epidural steroid injections
2- Pain management referral for narcotic
management of symptoms
3- Lumbar laminectomies at L4-5
4- Lumbar laminectomies and fusion at L4-5
5- Bilateral lumbar laminotomies at L3-4 and L4-
5

DISCUSSION: Patients with a degenerative


spondylolisthesis and severe stenosis who have failed
appropriate nonsurgical management are candidates for
surgical intervention. Most studies show good to
excellent results in more than 85% of patients after
lumbar decompression for stenosis. Atlas and associates
found that at 8- to 10-year follow-up, leg pain relief and
back-related functional status were greater in those
patients opting for surgical treatment of the stenosis. Similarly, the decision to fuse a
spondylolisthetic segment has been supported in the literature. Herkowitz and Kurz
compared decompressive laminectomy alone and decompressive laminectomy with
intertransverse arthrodesis in 50 patients with single-level spinal stenosis and degenerative
spondylolisthesis. They demonstrated good to excellent results in 90% of the fused group
compared to 44% in the nonfusion group. The decision to include instrumentation during the
fusion is more controversial. Whereas the use of instrumentation has shown to improve
fusion rates, it has not been conclusively shown to improve the overall clinical outcomes of
patients. PREFERRED RESPONSE: 4

53. A 29-year-old man reports a 2-week history of severe neck pain after being
struck sharply on the back of the head and neck while moving a refrigerator down a
flight of stairs. Initial evaluation in the emergency department revealed no obvious
fracture and he was discharged in a soft collar. Neurologic examination is within
normal limits, and radiographs taken in the office are shown in Figures 21a through
21c. Subsequent MRI scans show intra-substance rupture of the transverse atlantal
ligament. What is the most appropriate treatment option at this time?

32
AAOS 2009 Spine Self-Assessment Examination
1- Discontinue use of the soft collar and encourage range of motion
2- Semi-rigid collar immobilization for 6 to 8 weeks
3- Surgical stabilization
4- Halo skeletal fixation
5- Outpatient physical therapy with isometric neck exercises

DISCUSSION: Dickman and associates classified injuries of the transverse atlantal ligament
into two categories. Type I injuries are disruptions through the substance of the ligament
itself. Type II injuries render the transverse ligament physiologically incompetent through
fractures and avulsions involving the tubercle of insertion of the transverse ligament on the
C1 lateral mass. Type I injuries are incapable of healing without supplemental internal
fixation. Type II injuries can be treated with a rigid cervical orthosis with a success rate of
74%. Surgery may be required for type II injures that fail to heal with 3 to 4 months of
nonsurgical management. PREFERRED RESPONSE: 3

54. Figure 22 reveals what anatomic variant of the lumbar spine?

1- Spina bifida occulta


2- Unilateral sacralization
3- An aplastic or hypoplastic pedicle
4- Lumbarization
5- Facet tropism

DISCUSSION: Unilateral sacralization of the fifth lumbar


vertebra was first described by Bertolotti in 1917. Bertolotti’s
syndrome is present in 12% to 21% of the population. The
altered biomechanics have been postulated to cause low back
pain by placing increased stress on the adjacent cephalad
disk, thus contributing to accelerated degenerative disk
disease at this level. It has also been found that the
neoarticulation between the enlarged transverse process and the sacrum and/or ilium may be
a source of neural impingement on the exited L5 nerve root and results in radicular pain
syndrome. Brault and associates reported on a case treated surgically at the Mayo Clinic, in
which the pain generator was found to be the contralateral facet joint.PREFE RESPONSE: 2

55. Posterior lumbar spine arthrodesis may be associated with adjacent segment
degeneration cephalad or caudad to the fusion segment. Which of the following is
the predicted rate of symptomatic degeneration at an adjacent segment warranting
either decompression and/or arthrodesis at mid-range follow-up (5-10 years) after
lumbar fusion?

1- 2%
2- 10%
3- 25%
4- 50%
5- 80%
33
AAOS 2009 Spine Self-Assessment Examination
DISCUSSION: The rate of symptomatic degeneration at an adjacent segment warranting
either decompression or arthrodesis was predicted to be 16.5% at 5 years and 36.1% at 10
years based on a Kaplan-Meier analysis. PREFERRED RESPONSE: 3

56. A 24-year-old man who was involved in a high speed motor vehicle accident is
transferred for definitive care after having been diagnosed with an acute spinal cord
injury from a fracture-dislocation at C6-7. He has a complete C6 neurologic level and
it is now approximately 10 hours from his injury. What is the most appropriate
pharmacologic treatment at this time?

1- No pharmacologic intervention is recommended at this time


2- Administration of methylprednisolone with an initial bolus of 30 mg/kg
followed by 5.4 mg/kg for 24 hours
3- Administration of methylprednisolone with an initial bolus of 30 mg/kg
followed by 5.4 mg/kg for 48 hours
4- Administration of naloxone with an initial bolus of 30 mg/kg followed by
5.4 mg/kg for 24 hours
5- Administration of naloxone with an initial bolus of 30 mg/kg followed by
5.4 mg/kg for 48 hours

DISCUSSION: The standard practice in the pharmacologic treatment of a spinal cord injury in
the United States has been the administration of methylprednisolone with an initial bolus of
30 mg/kg followed by 5.4 mg/kg for 24 hours, in accordance with the findings of the second
and third National Acute Spinal Cord Injury Studies (NASCIS). Although the studies have
subsequently drawn criticism for their methodology and outcomes, it has been generally
accepted that beneficial neurologic outcomes were anticipated in patients who were able to
start the protocol within 8 hours of their initial injury. Further improvement was noted in
patients receiving the methylprednisolone within 3 hours of their injury and continuing an
infusion for 48 hours. In this patient, who is outside the 8-hour treatment window, no studies
have supported starting the methylprednisolone protocol at this time. PREFE RESPONSE: 1

57. Figures 23a and 23b show the MRI scans of a 50-year-old woman who has
increasing gait disturbance. She reports three falls in the past week. Examination
reveals hyperreflexia, motor weakness in the biceps and triceps, and a positive
Hoffman’s sign. What is the most appropriate treatment plan?

1-Observation
2-Physical therapy
3-Epidural steroid injections
4-Cervical laminectomy
5-Anterior cervical
diskectomy and fusion

34
AAOS 2009 Spine Self-Assessment Examination
DISCUSSION: The patient has obvious signs of progressive myelopathy. Based on her
significant physical examination findings, nonsurgical management will not significantly
impact her outcome. Cervical decompression alone is contraindicated in patients with
cervical kyphosis such as seen here. Anterior cervical fusion is the best option.
PREFERRED RESPONSE: 5

58. What structure (arrow) is shown in Figure 24?

1- Ilioinguinal nerve
2- Sympathetic chain
3- Ureter
4- Iliac vein
5- L5 nerve

DISCUSSION: The structure illustrated is the sympathetic


chain viewed from an anterolateral view of the lower
lumbar spine. It descends along the anterolateral aspect of
the spine into the pelvis closely adherent to the vertebral
column. The spinal nerves, including L5, can be seen
exiting from the foramen. The ureters descend from the
kidneys and cross anterior to the iliac vessels to the bladder. PREFERRED RESPONSE: 2

59. The best patient-related outcomes, following the surgical treatment of cauda
equina syndrome secondary to a large L5-S1 disk herniation, are most closely related
to which of the following?

1- Extent of bowel and bladder dysfunction


2- Extent of the motor deficit
3- Extent of the perianal saddle anesthesia
4- Timing of surgery
5- Location of the herniation

DISCUSSION: The most predictable positive outcome from spinal surgery due to a cauda
equina syndrome is early surgical intervention before any significant neurologic deficit
develops. Meta-analysis studies demonstrate that surgical intervention more than 48 hours
after the onset of cauda equina syndrome show an increased risk for poor outcomes.
PREFERRED RESPONSE: 4

60. A 45-year-old man undergoes an anterior cervical diskectomy and fusion at C5-
6 and C6-7 with instrumentation. During the first postoperative visit at 1 week, the
patient reports difficulty swallowing and mild anterior cervical tightness. The
anterior wound is benign and the patient denies any dyspnea or shortness of breath.
A postoperative radiograph is seen in Figure 25. What is the most appropriate
management at this time?

35
AAOS 2009 Spine Self-Assessment Examination
1-Admit for observation and reassurance
2-Surgical exploration and removal of the anterior
instrumentation
3-Esophageal swallowing study
4-Soft cervical collar and early range-of-motion exercises
5-CT of the cervical spine

DISCUSSION: The radiograph shows significant prevertebral


soft-tissue swelling following a two-level anterior cervical
diskectomy and fusion. The incidence of dysphagia 2 years
after anterior cervical spine surgery is 13.6%. Risk factors for
long-term dysphagia after anterior cervical spine surgery
include gender, revision surgeries, and multilevel surgeries.
The use of instrumentation, higher levels, or corpectomy
versus diskectomy did not significantly increase the prevalence of dysphagia. Lee and
associates demonstrated that while dysphagia after anterior cervical spine surgery is a
common early finding, it generally decreases significantly by 6 months with nonsurgical
management. A minority of patients experience moderate or severe symptoms by 6 months
after the procedure. Female gender and multiple surgical levels have been identified as risk
factors for the development of postoperative dysphagia.PREFERRED RESPONSE: 1

61. Steroids are thought to prevent neurologic deterioration after traumatic spinal
cord injury by which of the following mechanisms?

1- Maintains calcium influx into damaged cells


2- Destabilizes lysosomal membranes in the zone of injury
3- Reduces TNF-alpha expression
4- Increases NF-kB binding capacity
5- Maintains free radical oxidation

DISCUSSION: The proposed mechanisms by which steroids such as methylprednisolone are


thought to prevent neurologic deterioration by limiting secondary insult, include: decreasing
the area of ischemia in the cord, reducing TNF-alpha expression and NF-kB binding activity,
decreasing free radical oxidation and thus stabilizing cell and lysosomal membranes, and
checking the influx of calcium into the injured cells, thus reducing cord edema.
PREFERRED RESPONSE: 3

62. Which of the following mechanisms of inhibition has been linked to cigarette
smoking and lumbar spinal fusion?

1- Diminished revascularization of cancellous bone graft


2- Increased activity of osteoblasts
3- Increased activity of osteocytes
4- Antibody-induced necrosis
5- Inhibition of prostaglandins
36
AAOS 2009 Spine Self-Assessment Examination
DISCUSSION: Cigarette smoking has been directly linked to pseudarthrosis in spinal fusions.
The direct mechanism of action is diminished revascularization of cancellous bone graft.
Additionally, a smaller area of revascularization is seen in these grafts, as well as an
increased area of necrosis. Increased activity of osteoblasts would result in more bone
production. Increased activity of osteocytes would not affect the fusion because osteocytes
are mature bone cells. PREFERRED RESPONSE: 1

64. A previously healthy 29-year-old man reports a 2-day history of severe


atraumatic lower back pain. He denies any bowel or bladder difficulties and no
constitutional signs. Examination is consistent with mechanical back pain. No focal
neurologic deficits or pathologic reflexes are noted. What is the most appropriate
management?

1- Radiographs, including anterior, lateral, and oblique views


2- MRI of the lumbar spine and follow-up at the clinic in 1 week
3- Caudal epidural steroid injection
4- Reassurance, limited analgesics, and early range of motion as tolerated
5- Immediate MRI of the lumbar spine and possible urgent surgical
decompression

DISCUSSION: In general, a previously healthy patient with an acute onset of nontraumatic


lower back pain does not need diagnostic imaging before proceeding with therapeutic
treatment. In the absence of any “red flags” during the history and physical examination,
such as trauma or constitutional symptoms (ie, fevers, chills, weight loss), the appropriate
treatment for acute onset lower back pain is purely symptomatic treatment including limited
analgesics and early range of motion. Diagnostic imaging is not necessary unless the initial
treatment is unsuccessful and symptoms are prolonged. Miller and associates suggested
that the use of radiographs can lead to better patient satisfaction but not necessarily better
outcomes.PREFERRED RESPONSE: 4

65. Sacral fractures are most likely to be associated with neurologic deficits when
they involve what portion of the sacrum?

1- Zone 1 (the ala)


2- Zone 2 (the foramina)
3- Zone 3 (the central canal)
4- Zones 1 and 2
5- The sacral laminae

DISCUSSION: Denis divided the sacrum into three zones: zone 1 represents the lateral ala,
zone 2 represents the foramina, and zone 3 represents the central canal. A fracture is
classified according to its most medial extension. Those in zone 3 are typically bursting-type
fractures or fracture-dislocations and are most prone to neurologic sequelae.
PREFERRED RESPONSE: 3
37
AAOS 2009 Spine Self-Assessment Examination
66. Which of the following is associated with the use of bisphosphonates in the
setting of metastatic breast cancer to the spine?

1- Reduction in skeletal-related events by 30% to 40%


2- Jaw osteonecrosis in 15% of patients
3- Pain improvement in only 30% of patients
4- Improvement in serum hypocalcemia in 40% of patients
5- Accelerated bone destruction in 10% of patients

DISCUSSION: The indications of bisphosphonate therapy in breast cancer patients range


from the correction of hypercalcemia to the prevention of cancer treatment-induced bone
loss. Bisphosphonates reduce metastatic bone pain in at least 50% of patients and can
reduce the frequency of skeletal-related events by 30% to 40%. Osteonecrosis of the jaw
could occur in up to 2.5% of breast cancer patients during long-term bisphosphonate therapy.
PREFERRED RESPONSE: 1

67. A 67-year-old retired steelworker was involved in a motor vehicle accident and
sustained a midcervical spinal cord injury. Radiographs and MRI scans reveal severe
cervical stenosis and spondylosis without fractures or dislocations. Neurologic
examination reveals an ASIA C spinal cord impairment with greater motor
involvement of the upper extremities than the lower extremities. What is the
probability that the patient eventually will become ambulatory?
1- 2% to 5%
2- 15% to 20%
3- 35% to 45%
4- 60% to 70%
5- Greater than 90%

DISCUSSION: The patient sustained an incomplete spinal cord injury known as central cord
syndrome. Central cord syndrome characteristically has disproportionate involvement of the
upper extremities with the lower extremities being relatively spared. It is most commonly
seen after cervical injuries in elderly patients with spondylosis and spinal stenosis, often
without fracture. Penrod and associates noted that 23 of 59 patients with central cord
syndrome (ASIA C and D) ultimately walked. The poorest prognosis, however, was in ASIA
C patients older than age 50, in which only 40% walked. PREFERRED RESPONSE: 3

68. A 20-year-old man involved in a motor vehicle accident is brought to the


emergency department with a C6-7 unilateral facet dislocation. His neurologic
examination reveals a focal left-sided C7 nerve root palsy. He is awake and
cooperative with questioning and has no other obvious traumatic injuries. What is
the most appropriate treatment at this time?
1-Further imaging studies, including MRI
2-An awake closed reduction with Gardner-Wells traction with neurologic exam
3-Immobilization in a halo skeletal fixation for definitive treatment
4-Closed reduction under general anesthesia
5-Immediate open reduction and internal fixation in the surgical suite
38
AAOS 2009 Spine Self-Assessment Examination
DISCUSSION: In the patient who is neurologically intact or has an incomplete injury from a
cervical facet dislocation, a closed reduction with weighted tong traction is appropriate when
the patient is awake, alert, and cooperative. Although there is a risk that a cervical facet
dislocation could occur with an underlying cervical disk herniation, Vaccaro and associates
have shown that closed reduction can be safely carried out in the awake, responsive patient.
Closed reduction can be performed in the emergency department with traction with skull
tongs or a halo ring. A slow stepwise application of weight is added until a reduction is
achieved. Any worsening of the neurologic status of the patient requires immediate
termination of the closed reduction and further diagnostic imaging before proceeding with
further treatment.

PREFERRED RESPON: 2

69. A 66-year-old man reports a 2-week history of worsening low back and leg
pain. He reports that his pain is aggravated by lying down and relieved by standing
and walking. He notes that he has been losing weight recently and that his pain has
been awakening him during the night. His medical history is significant for
hypertension, coronary artery disease, and prostate cancer. His physical
examination is essentially unremarkable. Lumbar radiographs are within normal
limits. What is the most appropriate management for this patient?

1- MRI of chest
2- Laboratory studies, including a complete blood cell (CBC) count,
erythrocyte sedimentation rate (ESR), and urinalysis, PSA, CEA
3- Activity alterations to avoid undue back irritation
4- Comfort measures, including medications
5- Spinal manipulative therapy within the first 6 weeks

DISCUSSION: In the initial assessment of acute low back pain in adults, no diagnostic testing
is indicated during the first 4 weeks in the absence of “red flags” for a serious underlying
condition. The purpose of the initial assessment of acute low back pain in adults is to rule out
serious underlying conditions presenting as low back pain. The Agency for Healthcare Policy
and Research, in its 1994 clinical practice guideline, identified four serious conditions that
may present with low back pain, including fracture, tumor, infection, and cauda equina
syndrome. This patient has five “red flags” for a spinal tumor as a possible etiology of his low
back pain, including age of older than 50 years, constitutional symptoms (recent weight loss),
pain worse when supine, severe nighttime pain, and a history of cancer. Of these, his history
of cancer is most significant, as greater than 90% of spinal tumors are metastatic. In order of
frequency, breast, prostate, lung, and kidney make up approximately 80% of all secondary
spread to the spine. In the presence of “red flags” for tumor or infection, it is recommended
that the clinician obtain a CBC count, ESR, and a urinalysis. If these are within normal limits
and suspicions still remain, consider consultation or seek further evidence with a bone scan,
radiographs, or additional laboratory studies. Negative radiographs alone are insufficient to
rule out disease. If radiographs are positive, the anatomy can be better defined with MRI.

PREFERRED RESPONSE: 2

39
AAOS 2009 Spine Self-Assessment Examination
70. Which of the following increases radiation exposure to patients and personnel
during surgery?

1- Orienting the beam in the opposite direction of the working team and
keeping the team outside a 6-foot radius of the fluoroscopy machine
2- Orienting the cathode ray tube beneath the patient with the image
intensifier receptor as close to the patient as possible
3- Limiting the beam on time to only what is clinically important
4- The use of continuous fluoroscopy whenever possible to ensure proper
placement of implants
5- The use of lead glasses, a thyroid shield, and a lead apron with an
equivalent lead thickness of 0.25 mm

DISCUSSION: Continuous fluoroscopy and cine radiography expose the patient and
personnel to markedly increased levels of direct and scatter radiation exposure. Continuous
fluoroscopy should be limited to only what is absolutely needed for safe completion of the
procedure. By orienting the cathode ray tube beneath the patient and placing the image
intensifier as close as clinically possible to the patient, scatter radiation exposure to the
personnel is minimized. PREFERRED RESPONSE: 4

71. A 78-year-old woman undergoes her third lumbar decompression and fusion
from L3 to L5 without complication. On the morning of postoperative day 3,
examination reveals painless, flaccid weakness of both lower extremities. She also
has an absent bulbocavernous reflex and a mild saddle paresthesia. MRI scans of the
lumbar spine are shown in Figures 26a and 26b. What is the most appropriate
management at this time?

1- Continued serial neurologic examinations


2- CT with a myelogram of the lumbar spine
3- Immediate surgical exploration and hematoma drainage
4- Electromyography of bilateral lower extremities
5- IV antibiotics for 24 hours, followed by surgical exploration if symptoms persist

40
AAOS 2009 Spine Self-Assessment Examination
DISCUSSION: The MRI scans reveal a large postoperative hematoma causing significant
thecal compression. An epidural hematoma with neurologic deficit is a surgical emergency
requiring immediate evacuation of the hematoma. Although the incidence of postoperative
epidural hematomas is rare, the consequences of a missed diagnosis can be catastrophic.
Early recognition and evacuation are essential in preserving or restoring neurologic function.
Uribe and associates attributed delayed postoperative hematomas to previous multiple
lumbar surgeries as a possible contributing factor. PREFERRED RESPONSE: 3

72. Figures 27a through 27c show the radiographs and CT scan of a 27-year-old
man who sustained a low-velocity gunshot wound to the neck. He is quadriplegic
(ASIA A), hemodynamically stable, and does not have drainage from his wound.
After initial resuscitation and stabilization, the cervical spine and spinal cord injuries
are best managed by

1- wound debridement, anterior corpectomy, spinal cord decompression, dural


repair, and anterior fusion with strut graft and anterior plating.
2- wound debridement, anterior corpectomy, spinal cord decompression, dural
repair, anterior fusion with strut graft and anterior plating followed by posterior
laminectomy, and spinal cord decompression and dural repair with excision of the
bullet fragment.
3- wound debridement, anterior corpectomy, spinal cord decompression, dural
repair, anterior fusion with strut graft and anterior plating followed by laminectomy
and posterior fusion, and spinal cord decompression and dural repair with excision of
the bullet fragment.
4- laminectomy and posterior fusion, and spinal cord decompression and dural
repair with excision of the bullet fragment.
5- surgical treatment based on extraspinal pathology with orthotic treatment of
the spinal fractures.

DISCUSSION: Although the spinal canal has been penetrated, the lateral masses are intact
bilaterally with only partial destruction of the vertebral body and penetration of the lamina on
one side, thus the cervical spine is not unstable and surgical stabilization is not indicated.
Dural repair is not indicated since there is no external cerebrospinal fluid leakage. Surgical
treatment should be based on the need to treat extraspinal pathology only.
PREFERRED RESPONSE: 5
41
AAOS 2009 Spine Self-Assessment Examination
73. Which of the following is a true statement regarding thoracic disk herniations?

1- Are most commonly discovered during the fifth to seventh decades of


life
2- Occur with similar frequency as cervical disk herniations
3- Occur most commonly in the midthoracic or apical region of the spine
4- Can be found in 40% of asymptomatic individuals
5- Are best treated surgically with posterior laminectomy and excision

DISCUSSION: Symptomatic herniations of the thoracic spine are much less common than
those of the cervical or lumbar region. They tend to occur most commonly during the third to
fifth decades of life and although they can be found at all levels, they are most common in the
lower third near the thoracolumbar region. Posterior laminectomy and disk excision has the
highest rate of neurologic deterioration and is not recommended. Multiple studies have
shown that herniated thoracic disks can be found at one or more levels in 40% of
asymptomatic individuals. PREFERRED RESPONSE: 4

74. A sentinel event is defined as an unexpected occurrence involving death or


serious physical or psychological injury, or the risk thereof. What is the most
common sentinel event related to spine surgery?

1- Surgery on the wrong patient


2- Surgery on the wrong side
3- Incorrect procedure performed
4- Intraoperative death
5- Surgery on the wrong level

DISCUSSION: Patient safety and prevention of medical errors is a major focus of recent
national advocacy groups. Analysis has shown that the most common sentinel event in spine
surgery is surgery on the wrong level. Therefore, it is recommended that every patient have
the surgical site signed, the level of surgery marked intraoperatively, and a radiograph taken.
Surgery on the wrong level is most likely to occur in single-level decompressive procedures.
PREFERRED RESPONSE: 5

75. What structure is most at risk with anterior penetration of C1 lateral mass
screws?

1- Vertebral artery
2- External carotid artery
3- Internal carotid artery
4- Pharynx
5- Glossopharyngeal nerve

42
AAOS 2009 Spine Self-Assessment Examination
DISCUSSION: Posterior screw fixation of the upper cervical spine has gained a great deal of
popularity due to its stable fixation, obviating the use of halo vest immobilization, and its high
fusion rates. The use of screws in this location, however, has introduced a whole new set of
potential complications. Vertebral artery injury is one of the most feared complications
associated with screws in the C1/C2 region. This structure, however, is lateral and posterior
at the C2 level and then penetrates the foramen transversarium of C1 to lie cephalad to the
arch of C1 before entering the foramen magnum. It is the internal carotid artery that lies
immediately anterior to the arch of C1 that is particularly at risk by anterior penetration of C1
lateral mass or C1-C2 transarticular screws as demonstrated by Currier and associates. The
internal carotid artery lies posterior to the pharynx. The external carotid artery and the
glossopharyngeal nerve are not at risk with this method of fixation.

PREFERRED RESPONSE: 3

76. During the application of halo skeletal fixation, the most appropriate position
for the placement of the anterior halo pins is approximately 1 cm above the superior
orbital rim and

1- lateral placement, directly within the temporalis muscle.


2- lateral to the medial third of the orbit.
3- lateral to the lateral aspect of the orbit.
4- above the medial third of the orbit.
5- lateral between the temporalis muscle and zygomatic temporal nerve.

DISCUSSION: Halo fixation is the most rigid form of cervical orthosis, but complications can
arise from improper placement of the initial halo ring. A relatively safe zone for anterior pin
placement is located 1 cm above the orbital rim and superior to the lateral two thirds of the
orbit. This position avoids the supraorbital and supratrochlear nerves and arteries over the
medial one third of the orbit. The more lateral positions in the temporal fossa have very thin
bone and can interfere with the muscles of mastication. Posterior pin site locations are less
critical; positioning on the posterolateral aspect of the skull, diagonal to the contralateral
anterior pins, is generally desirable.

PREFERRED RESPONSE: 2

43
AAOS 2009 Spine Self-Assessment Examination
77. Figures 28a and 28b show the sagittal and axial lumbar MRI scans of a 72-year-
old man who reports dull aching back pain that spreads to his legs, calves, and
buttocks. He has had the pain for several years and it is precipitated by standing and
walking and relieved by sitting. His symptoms have been worsening over the past
year and he notes that he is leaning forward while walking to help relieve his
symptoms. He has had no treatment to date. What is his prognosis if he chooses to
pursue nonsurgical management for this condition?

1- He can expect complete resolution of his symptoms during the first month.
2- All patients experience relief within 3 months and continue to improve over
the next 4 years.
3- Most patients experience some pain relief within the first 3 months.
4- He may experience some improvement but if he requires surgery at a later
date he will have a poorer result because of the delay.
5- The patient requires immediate surgery to avoid permanent nerve damage.

DISCUSSION: The patient has lumbar spinal stenosis and the MRI scans reveal the
pathology at L4-5, which is secondary to posterior disk bulging and hypertrophy and infolding
of the ligamentum flavum, as well as degenerative facet arthrosis. The degree of spinal
stenosis is moderate and his symptoms are positional in nature. Tadokoro and associates
reported on a prospective study of 89 patients older than 70 years of age who underwent
nonsurgical management for lumbar spinal stenosis. They found the prognosis to be
relatively good with patients scoring at “excellent” or “good” for activities of daily living at final
follow-up. However, they did note that patients with a complete block on myelography did not
respond favorably to nonsurgical management. Amundsen and associates reported on a 10-
year prospective study comparing surgical care to nonsurgical management. They
concluded that, while the long-term results largely favored surgical treatment, more than half
of the nonsurgically managed patients had a satisfactory outcome. They also concluded that
a delay of surgery for some months did not worsen the prognosis. Therefore, their
recommendation was for an initial primarily nonsurgical approach.

PREFERRED RESPONSE: 3

44
AAOS 2009 Spine Self-Assessment Examination
78. Which of the following vertebrae has the smallest pedicle isthmic width in a
nondeformity patient?

1- T10
2- T11
3- T12
4- L1
5- L2

DISCUSSION: The smallest pedicle isthmic width is at L1, whereas T12 has the largest
pedicle width in the upper lumbar and lower thoracic spine. Although smaller in diameter
than T12, both T10 and T11 have larger pedicle widths than L1. PREFERRED RESPON: 4

79. Which of the following represents a contraindication for interspinous process


decompression for the treatment of lumbar spinal stenosis?

1- Grade I degenerative spondylolisthesis


2- Inability to walk at least 100 feet
3- Cauda equina syndrome
4- Fixed sensory deficit
5- Intermittent foot drop

DISCUSSION: Kondrashov and associates noted stable good outcomes at 4 years in 14 of


18 patients treated with X-STOP interspinous process decompression as defined as an
improvement over preoperative Oswestry scores of 15 points or more. Similar results were
seen after 1 year in a European study by Siddiqui and associates. Exclusion and inclusion
criteria for these studies varied somewhat, but cauda equina syndrome was the only
exclusion criteria listed in both studies. All of the other choices did not represent exclusion
criteria in either study. PREFERRED RESPONSE: 3

80. Which of the following statements about hoarseness due to vocal cord
paralysis after anterior cervical diskectomy and fusion is most accurate?

1- Vocal cord paralysis is three times as likely with a right-sided approach as


compared to a left-sided approach.
2- Vocal cord paralysis is twice as likely with a right-sided approach as compared
to a left-sided approach.
3- Vocal cord paralysis is equally likely with either a right-sided or a left-sided
approach.
4- Vocal cord paralysis is three times as likely with a left-sided approach as
compared to a right-sided approach.
5- Vocal cord paralysis is twice as likely with a left-sided approach as compared to
a right-sided approach.

45
AAOS 2009 Spine Self-Assessment Examination
DISCUSSION: It has been traditionally taught that a left-sided approach to the anterior
cervical spine is associated with a lower incidence of injury compared to the right-sided
approach. This is due in part to the anatomic differences in the path the recurrent laryngeal
nerve (RLN) takes on the right as compared to the left. Both nerves ascend in the
tracheoesophageal groove after branching off the vagus nerve in the upper thorax. The left-
sided RLN loops around the aortic arch and stays relatively medial as compared to the right-
sided RLN which loops around the right subclavian artery and is somewhat more lateral at
this point, and therefore is theoretically more vulnerable as it ascends toward the larynx
before becoming protected in the tracheoesophageal groove. Furthermore, the variant of a
nonrecurrent inferior laryngeal nerve branching directly off the vagus nerve at the level of the
midcervical spine is much more common on the right than the left. Despite this reasoning,
there has been no clinical evidence to suggest that laterality of approach for anterior cervical
surgery makes any difference in the incidence of vocal cord paralysis. Furthermore, two
recent studies have shown that the incidence of RLN injury and vocal cord paralysis is equal
with either side of approach. PREFERRED RESPONSE: 3

81. A 23-year-old man is involved in a motor vehicle accident. An AP radiograph is


shown in Figure 29a, and axial and sagittal CT scans are shown in Figures 29b and
29c. Neurologic examination shows 1/5 strength of his quadriceps and iliopsoas on
the right, with 1/5 quadriceps function on the left. Definitive treatment of his injury
should consist of

1- anterior corpectomy with interbody strut.


2- posterior fusion with instrumentation and posterolateral
decompression.
3- closed reduction and a thoracolumbosacral orthosis (TLSO).
4- anterior reduction and instrumentation.
5- supine bed rest for 6 weeks, followed by immobilization in a TLSO.

DISCUSSION: The imaging studies show a fracture-dislocation. Surgical treatment of this


injury consists of a decompression reduction, stabilization, and fusion. A posterolateral
decompression can also be performed as necessary. An isolated anterior procedure in this
type of injury is contraindicated. The anterior longitudinal ligament is most likely intact;
therefore, an anterior procedure further destabilizes the spine. Reduction by an anterior
approach would also be difficult. Nonsurgical management of the neurologic injury in this
patient is not indicated.PREFERRED RESPONSE: 2

46
AAOS 2009 Spine Self-Assessment Examination
82. Surgical treatment for symptomatic disk herniations is associated with which
of the following?

1- Substantial rate of nerve root injury


2- Early relief of pain sustained out to 2 years
3- Recurrent herniation rate of 35%
4- Outcomes that are substantially worse than nonsurgical management
5- 10% rate of infectious diskitis

DISCUSSION: The recently published SPORT trial verifies that surgical treatment of
symptomatic disk herniations is associated with early and sustained pain relief. The trial also
verifies that nonsurgical management is associated with improved symptoms as well. Nerve
root injury, recurrent herniation, and diskitis are known complications of surgery, but all are
less common than described above.

PREFERRED RESPONSE: 2

83. A 25-year-old man is unresponsive at the scene of a high-speed motor vehicle


accident and remains obtunded. Initial evaluation in the emergency department
reveals a left-sided femoral shaft fracture and a right-sided humeral shaft fracture.
The cervical spine remains immobilized in a semi-rigid cervical collar, and the initial
AP and lateral radiographs obtained in the emergency department are
unremarkable. What is the most appropriate management at this time?

1- Lateral radiographs with passive flexion/extension views


2- Helical CT scan of the cervical-thoracic region
3- Careful manual palpation of the cervical spine for subtle defects or step-
offs
4- MRI of the cervical spine
5- Continued use of the cervical collar until the patient becomes responsive
for examination

DISCUSSION: Clearance of the cervical spine can be difficult in the obtunded or


unresponsive patient. Various trauma series have been reported to detect up to 95% of
cervical fractures but only when ideal imaging views have been obtained, which is not often
possible in the unresponsive or uncooperative patient. Passively performed cervical flexion-
extension under live fluoroscopy has been suggested but is not without inherent risk in the
potentially unstable cervical spine. CT of the cervical spine has gained acceptance for the
evaluation of these patients given the excellent evaluation of the osseous anatomy and for
the common availability in most emergency departments. Sanchez and associates, using a
protocol to evaluate for cervical spine injuries after blunt trauma, were able to detect 99% of
cervical fractures with 100% specificity. PREFERRED RESPONSE: 2
47
AAOS 2009 Spine Self-Assessment Examination
84. A 55-year-old woman undergoes an anterior cervical diskectomy and fusion at
C5-C6 through a left-sided approach. One year later, she requires an anterior cervical
diskectomy and fusion on another level. Which of the following is considered a
contraindication to performing a right-sided approach for the revision procedure?

1- Revision surgery caudad to C6


2- Persistent left cervical radiculopathy
3- History of a left-sided Horner’s syndrome
4- Transient dysphagia following the initial anterior cervical procedure
5- Nonfunctional left vocal cord

DISCUSSION: When attempting a revision anterior cervical approach from the side opposite
the original approach, it is important to evaluate the function of the vocal cords. If this
evaluation reveals dysfunction of the vocal cord on the side of the original approach, then an
approach on the contralateral side should not be attempted. Injury to the stellate ganglion,
which causes a Horner’s syndrome, should not preclude an approach on the contralateral
side. While the side of the symptomatology can influence the surgeon’s choice as to the side
of an anterior approach, it does not preclude a certain approach. When approaching the
lower cervical spine from the right side, the recurrent laryngeal nerve can cross the surgical
field and should be preserved. Excessive intraoperative pressure on the esophagus can
increase the incidence of dysphagia, but its incidence is no different with either approach.
PREFERRED RESPONSE: 5

85. A 56-year-old woman sustained the fracture shown in Figures 30a and 30b in a
motor vehicle accident. What mechanism is most likely responsible for the injury?

1- Flexion distraction
2- Vertical shear
3- Extension distraction
4- Flexion compression
5- Axial load
48
AAOS 2009 Spine Self-Assessment Examination
DISCUSSION: The CT scans show a burst fracture that results from an axial load injury. The
radiographic hallmark of a burst fracture is compression of the posterior cortex of the
vertebral body with retropulsion of bone into the spinal canal. AP radiographs often show
widening of the interpedicular distance with a fracture of the lamina. PREFER RESPONSE: 5

86. In providing culturally competent care to a Muslim woman with a cervical


spine injury, which of the following most accurately describes the steps a male
orthopaedist should take to respect her religious beliefs during his examination?

1- No one should be in the exam room except the patient and the physician.
2- Another woman should be in the exam room and only the affected body part
should be exposed.
3- A chaperone of either gender should be in the exam room and no skin should
be exposed.
4- No particular steps need to be taken in this case.
5- The patient’s closest male relative should be in the exam room but a standard
hospital gown may be used.

DISCUSSION: In examining a traditional Muslim woman, a male physician should have


another woman present, and the patient’s husband, if possible. Only the affected limb or
area needing examination should be exposed. PREFERRED RESPONSE: 2

87. Figure 31 shows the radiograph of a 64-year-old woman who is seen in the
emergency department following a motor vehicle accident. She has no voluntary
motor function in her distal upper extremities or lower extremities. She does not
have a bulbocavernosus reflex. She has a blood pressure of 80/50 mm Hg with a
pulse of 50/min. Her hypotension does not improve with initial fluid resuscitation.
Further treatment of her hypotension should consist of

1-continued rapid fluid infusion.


2-administration of broad-spectrum antibiotics.
3-administration of 30/mg/kg methylprednisolone
over 1 hour.
4-administration of pressors.
5-cardioversion and implantation of a pacemaker.

DISCUSSION: The hallmark of neurogenic shock is


hypotension without tachycardia. It is associated most
commonly with high cervical spinal cord injuries and
results from loss of function of the sympathetic nervous
system. Because the peripheral vasculature is dilated due
to loss of its sympathetic tone, continued rapid
administration of fluid corrects the hypotension and can
quickly lead to fluid overload and congestive heart failure.
49
AAOS 2009 Spine Self-Assessment Examination
Therefore, neurogenic shock is best treated by the use of pressors. Cardioversion or
administration of antibiotics or systemic steroids is not appropriate treatment for this patient’s
hypotension. PREFERRED RESPONSE: 4

88. What is the typical axial plane transverse angulation of the thoracic pedicles?

1- 5 degrees medial at T1 and T2; 10 degrees from T3 to T10


2- 5 degrees lateral at T1; neutral at T2; 5 degrees medial from T3 to T12
3- 10 degrees medial from T1 to T10; 15 degrees medial at T11 and T12
4- 10 degrees medial from T1 to T12
5- 25 degrees medial at T1; 15 degrees at T2; and 10 degrees medial from T3 to
T10

DISCUSSION: Thoracic pedicles typically are angled 25 degrees medially at T1 so the


starting point is more lateral. T2 angles about 15 degrees, and then the pedicles average
about 5 to 7 degrees down to T10. At T11 and 12, the angulation is minimal.
PREFERRED RESPONSE: 5

89. What muscle is most often encountered during surgical approaches to C5-6?

1- Omohyoid
2- Cricohyoid
3- Splenius capitus
4- Thyrohyoid
5- Posterior digastric

DISCUSSION: The omohyoid muscle crosses the surgical field from inferior lateral to anterior
superior traveling from the scapula to the hyoid bone and may need to be transected. The
posterior digastric crosses the field as well but higher near C3-4. The other muscles run
longitudinally. PREFERRED RESPONSE: 1

90. Which of the following lumbar disk components has the highest tensile
modulus to resist torsional, axial, and tensile loads?

1- Nucleus pulposus
2- Cartilaginous end plate
3- Anterior longitudinal ligament
4- Annulus fibrosis
5- Cellular matrix

DISCUSSION: The annulus fibrosis has a multilayer lamellar architecture mode of type I
collagen fibers. Each successive layer is oriented at 30 degrees to the horizontal in the
opposite direction, leading to a “criss-cross” type pattern. This composition allows the
annulus, which has the highest tensile modulus, to resist torsional, axial, and tensile
loads.PREFERRED RESPONSE: 4
50
AAOS 2009 Spine Self-Assessment Examination
91. When comparing the overall outcomes of surgical versus nonsurgical
treatment of stable thoracolumbar burst fractures in patients without neurologic
injury, 5 years following injury, the principle differences lie in

1- fracture kyphosis.
2- reduction of retropulsed bone.
3- pain reduction.
4- incidence of complications.
5- return to work.

DISCUSSION: When patients are compared at 5 years follow-up, there are no statistically
significant differences between the two groups with respect to kyphosis, the degree of
retropulsed bone resorption, pain and function levels, or the ability to return to work.
Nonsurgical management of stable neurologically intact burst fractures has a very low
incidence of complications. PREFERRED RESPONSE: 4

92. A 42-year-old woman who has had an 18-month history of severe low back
pain is referred to your office for surgical evaluation. She reports that the pain
initially began with right lower extremity pain and management consisted of oral
analgesics, nonsteroidal anti-inflammatory drugs, and muscle relaxants. She has
seen a chiropractor as well as a pain management specialist and she is status-post
epidural steroid injections. She has also completed exhaustive physical therapy, as
she is a certified athletic trainer and runs a health fitness program at a community
hospital. Currently, she denies lower extremity pain and her pain is isolated to her
low back and is subjectively graded as 8/10, with 10 being the worst pain she has
ever experienced. The pain is interfering with her activities of daily living and she is
seeking definitive treatment. Figures 32a through 32c show current MRI scans.
Based on the current available medical literature, what is the most appropriate
treatment?

51
AAOS 2009 Spine Self-Assessment Examination
1- Continued nonsurgical management to include long-acting narcotic analgesics
2- Referral for vertebral axial decompression
3- Referral to interventional pain management for a spinal cord stimulator
4- Intradiskal electrothermal therapy (IDET) at L5-S1
5- Lumbar spinal fusion at L5-S1

DISCUSSION: The MRI scans reveal advanced degenerative disk disease at L5-S1.
Nonsurgical management has failed to provide relief and the patient is quite debilitated as a
result of her back pain. Fritzell and associates demonstrated that in a well-informed and
selected group of patients with severe low back pain, lumbar fusion can diminish pain and
decrease disability more efficiently than commonly used nonsurgical treatments. In a recent
updated Cochrane Review of surgery for degenerative lumbar spondylosis, it was noted that
while Fritzell and associates appeared to provide strong evidence in favor of fusion, a more
recent trial by Brox and associates demonstrated no difference between those patients
undergoing lumbar fusion compared to those receiving cognitive intervention and exercise.
The Cochrane Review suggests that this may reflect a difference between the control groups.
Fritzell and associates compared lumbar fusion to standard 1990s “usual care,” whereas
Brox and associates compared lumbar fusion to a “modern rehabilitation program.” Bear in
mind that this patient is a certified athletic trainer and runs a hospital health fitness
department; therefore, at least for purposes of this question, it can be assumed that she has
participated in a “modern rehabilitation program.” The Cochrane Review goes on to state
that preliminary results of three small trials of intradiskal electrotherapy suggest that it is
ineffective and that preliminary data from three trials of disk arthroplasty do not permit firm
conclusions. PREFERRED RESPONSE: 5

93. Figure 33 shows the MRI scan of a 55-year-old woman who has had a 6-week
history of back and leg pain. Which of the following clinical scenarios is most
consistent with the MRI scan findings at L4-L5?

1-L4 nerve root radiculopathy


2-L5 nerve root radiculopathy
3-Associated bowel and bladder dysfunction
4-Symptoms associated with arachnoiditis
5-Wide-based gait, left-sided Hoffman’s sign

DISCUSSION: The MRI scan reveals a L4-L5 foraminal disk herniation originating from the
L4-5 disk space that has migrated up into the foramen, compressing the left L4 nerve root.
There is normal distribution of the roots in the cerebrospinal fluid, excluding arachnoiditis as a
diagnosis, and disk herniation in this location would not result in cauda equina syndrome or
myelopathy. PREFERRED RESPONSE: 1

52
AAOS 2009 Spine Self-Assessment Examination
94. Intradiskal electrothermal therapy (IDET) uses an intradiskal catheter to deliver
controlled thermal energy to the inner periphery of the annulus fibrosis of a
chronically painful intervertebral disk. Lumbar diskography is used diagnostically to
identify the presumed pain generator to be targeted with IDET. Based on the
medical literature, what can be said about the current status of IDET?

1- IDET has been proven to seal annular tears in the annulus fibrosis.
2- IDET restores segmental stability by shrinking collagen fibrils in the disk.
3- IDET has demonstrated no significant benefit over placebo in controlled trials.
4- IDET is an unsafe procedure with significant risk of permanent complications.
5- IDET has demonstrated poor clinical results in all reported series to date.

DISCUSSION: Intradiskal electrothermal therapy (IDET) initial clinical results were reported in
2000. The early case series were quite encouraging with reported therapeutic success rates
of 60% to 80%. Early enthusiasm was high as IDET provided a nonsurgical treatment option
for an otherwise complex and difficult clinical entity, chronic diskogenic low back pain. The
actual mechanism of action was not well understood, and while the theoretic explanation
made good sense, it did not hold up under laboratory testing. Soon clinical results from the
field did not meet the high expectations set by the developers of the technique. Since those
early case studies, a few level I evidence studies have been conducted, one by Freeman and
associates and one by Pauza and associates. These randomized, placebo-controlled trials
demonstrated no significant benefit of IDET over the placebo. PREFERRED RESPONSE: 3

95. A 56-year-old mechanic has had pain in the hypothenar region of his dominant
right hand for the past 6 months. He reports weakness in his grip and pain is worse
with activity. Which of the following examination findings is most suggestive of a
cervical etiology?

1- Relief of symptoms with shoulder abduction (placing hand over the


head)
2- Hypothenar atrophy
3- Reproduction of pain with hyperflexion and contralateral rotation of the
head
4- Positive Tinel’s sign at the levator scapulae
5- Subluxable ulnar nerve at the cubital tunnel

DISCUSSION: Hypothenar atrophy is a nonspecific sign that can be seen in ulnar


neuropathy, C8 radiculopathy, or even cervical myelopathy; however, the atrophy usually is
not unilateral and includes other muscle groups. The Spurling test is an excellent method of
eliciting cervical radicular pain but involves hyperextension and ipsilateral rotation of the
cervical spine, resulting in nerve root compression by reducing the cross-sectional area of the
ipsilateral neuroforamen. Tinel’s sign at the levator scapulae, if present, is indicative of an
upper cervical (C3 or C4) radiculopathy. A subluxable ulnar nerve at the cubital tunnel, while
often asymptomatic, points toward cubital tunnel syndrome as an etiology for this patient’s
pain. The shoulder abduction relief (SAR) sign (relief of upper extremity pain with shoulder
53
AAOS 2009 Spine Self-Assessment Examination
abduction) is virtually pathognomic of cervical radiculopathy because this maneuver results in
relaxation of a compressed and/or inflamed cervical nerve root. The SAR sign is the
converse analog of the straight leg raising sign in the lumbar examination for lumbar
radiculopathy, as it relieves tension in the nerve root, thereby relieving symptoms.

PREFERRED RESPONSE: 1

96. A 35-year-old woman reports an 8-week history of neck pain radiating to her
right upper extremity. She denies any history of trauma or provocative event.
Examination reveals decreased pinprick sensation in her right middle finger,
otherwise sensation is intact bilaterally. Finger flexors and interossei demonstrate
5/5 motor strength bilaterally. Finger extensors are 4/5 on the right and 5/5 on the
left. The triceps reflex is 1+ on the right and 2+ on the left. The most likely diagnosis
is a herniated nucleus pulposus at what level?

1- C3-4
2- C4-5
3- C5-6
4- C6-7
5- C7-T1

DISCUSSION: The patient’s neurologic examination is consistent with a C7 radiculopathy on


the right side. In a patient with this symptom complex in the absence of trauma, a cervical
disk herniation is the most common etiology for a C7 radiculopathy. There are eight cervical
nerve roots and the C7 nerve exits at the C6-7 disk space and is most frequently impinged by
a disk herniation at this level.

PREFERRED RESPONSE: 4

97. What is the most common nonanesthetic-related reversible cause of changes


in intraoperative neurophysiologic monitoring data?

1- Pedicle screw misplacement


2- Patient positioning
3- Spinal cord ischemia
4- Retractor placement
5- Hypotension

DISCUSSION: Patient positioning that results in local nerve compression, plexus traction, or
improper neck alignment is the most common nonanesthetic-related cause of changes in
intraoperative neurophysiologic monitoring data during spinal surgery.

PREFERRED RESPONSE: 2

54
AAOS 2009 Spine Self-Assessment Examination
98. During a left-sided transforaminal lumbar interbody fusion at the L4-5 level,
the surgeon notes a significant amount of bleeding that cannot be controlled while
using a pituitary rongeur. What anatomic structure has been injured?

1- Aorta
2- Common iliac artery
3- Common iliac vein
4- External iliac artery
5- External iliac vein

DISCUSSION: The surgeon perforated the anterior longitudinal ligament and injured the
common iliac artery. Bingol and associates described injuries to the vascular structures
during lumbar disk surgery. The common iliac artery was most commonly affected and
constituted 76.9% of injuries.

PREFERRED RESPONSE: 2

99. Six weeks after onset, what is the most clearly accepted indication for surgical
management for lumbar disk herniation?

1- Stable sensory loss


2- Stable motor weakness
3- Refractory radicular pain
4- Size of the herniation
5- Lost time at work

DISCUSSION: In the absence of a cauda equina syndrome or progressive weakness, the


best indication for surgical management is refractory radicular pain. Surgical decision-
making should not be based on the size of the herniation. Large extruded herniations tend to
resolve more predictably than smaller herniations. Stable motor weakness and numbness
resolve similarly in both surgical and nonsurgical management, although surgery hastens the
process. When intractable radicular pain is the strict indication for surgery, surgical
intervention provides substantial and more rapid pain relief than nonsurgical care.

PREFERRED RESPONSE: 3

55
AAOS 2009 Spine Self-Assessment Examination
100. A 45-year-old woman has idiopathic scoliosis. Surgery is to include an anterior
thoracic release through an open left thoracotomy. The thoracotomy will have what
effect on the patient’s pulmonary function postoperatively?

1-Unaffected
2-Transiently reduced postoperatively but ultimately improves to greater than
preoperative function
3-Transiently reduced immediately postoperatively but then quickly returns to
preoperative levels
4-Improves postoperatively due to correction of the scoliosis and is maintained long
term
5-Reduced postoperatively and often remains reduced long term

DISCUSSION: A thoracotomy in an adult with idiopathic scoliosis causes a reduction in


pulmonary function that often does not return to preoperative levels. What pulmonary
function that does recover, recovers over many months. Long-term improvement in
pulmonary function, compared to preoperative function, is rarely seen. This should be
considered in planning surgical intervention in adults with scoliosis.

PREFERRED RESPONSE: 5

56

You might also like